इंदौर शाखा: IAS और MPPSC फाउंडेशन बैच-शुरुआत क्रमशः 6 मई और 13 मई   अभी कॉल करें
ध्यान दें:

प्रारंभिक परीक्षा


प्रारंभिक परीक्षा

प्रारंभिक परीक्षा, 2019: एक विश्लेषणात्मक अध्ययन

  • 05 Jul 2019
  • 172 min read

प्रिय अभ्यर्थियों,

2 जून, 2019 को आयोजित हुई UPSC की प्रारंभिक परीक्षा के प्रथम प्रश्नपत्र (सामान्य अध्ययन-I) का स्तर लगभग पिछले के समान ही देखने को मिला। जहाँ एक ओर पारंपरिक विषयों से कुछ जटिल प्रश्नों (इन प्रश्नों का स्वरुप काफी अस्पष्ट था) को प्रश्नपत्र में शामिल किया गया, वहीं दूसरी ओर पिछले दो वर्षों के करेंट अफेयर्स से भी प्रश्न पूछे गए। संपूर्ण प्रश्नपत्र का आकलन, विस्तृत विश्लेषण एवं अध्ययन करने के बाद हमें यह बताने में काफी हर्ष हो रहा है कि हमारी वेबसाइट, दृष्टि करेंट अफेयर्स टुडे मैगज़ीन, प्रारंभिक परीक्षा टेस्ट सीरीज़ ने प्रश्नपत्र का 80% भाग कवर किया है। इसमें से तक़रीबन 50% प्रश्न हमारी website में कवर हुए है।


सामान्य अध्ययन प्रश्नपत्र-I (SET A)

1. मुगल भारत के संदर्भ में, जागीरदार और ज़मींदार के बीच क्या अंतर है/हैं?

1. जागीरदारों के पास न्यायिक और पुलिस दायित्वों के एवज़ में भूमि आवंटनों का अधिकार होता था, जबकि ज़मींदारों के पास राजस्व अधिकार होते थे तथा उन पर राजस्व उगाही को छोड़कर अन्य कोई दायित्व पूरा करने की बाध्यता नहीं होती थी।
2. जागीरदारों को किये गए भूमि आवंटन वंशानुगत होते थे और ज़मींदारों के राजस्व अधिकार वंशानुगत नहीं होते थे।

नीचे दिये गए कूट का प्रयोग कर सही उत्तर चुनिये।

(a) केवल 1       

(b) केवल 2

(c) 1 और 2 दोनों

(d) न तो 1 और न ही 2

उत्तर: (d)

व्याख्या: मुगलकालीन भारत में, मनसबदारी प्रथा के अंतर्गत जागीरदारों के पास न्यायिक और पुलिस दायित्वों के एवज़ में भूमि आवंटनों का अधिकार होता था, जबकि ज़मींदारों के पास राजस्व अधिकार होते थे तथा इसके अलावा ज़मींदारों को सैन्य कर्तव्यों का निर्वहन भी करना पड़ता था और खिदमत का प्रदर्शन करना पड़ता था जिसमें, आवश्यकता के समय शाही सेना के लिये सैनिक उपलब्ध कराना और शाही दरबार में उपहार देना आदि शामिल था।

जागीरदारों को अक्सर पदोन्नति अथवा पदावनति के रूप में स्थानांतरित किया जाता था और यह मुग़ल साम्राज्य की कार्यप्रणाली एक नियमित हिस्सा था क्योंकि सम्राट के प्रति वफादारी, काफी हद तक एक कुलीन की जागीर या उसके मनसब के पद पर निर्भर थी; दूसरी ओर, ज़मींदारों के पास राजस्व के  वंशानुगत अधिकार थे क्योंकि ग्रामीण समाज पर उनका प्रभुत्त्व सभी पहलुओं में व्यापक था जो पैतृक और वंशानुगत संरक्षण पर आधारित था।


2. स्वतंत्र भारत में भूमि सुधारों के संदर्भ में, निम्नलिखित में से कौन-सा कथन सही है?

(a) हदबंदी कानून पारिवारिक जोत पर केंद्रित थे, न कि व्यक्तिगत जोत पर।

(b) भूमि सुधारों का प्रमुख उद्देश्य सभी भूमिहीनों को कृषि भूमि प्रदान करना था।

(c) इसके परिणामस्वरूप नकदी फसलों की खेती, कृषि का प्रमुख रूप बन गई।

(d) भूमि सुधारों ने हदबंदी सीमाओं को किसी भी प्रकार की छूट की अनुमति नहीं दी।

उत्तर: (b)

व्याख्या: भूमि सुधार कृषि काश्तकारों की आर्थिक स्थिति में सुधार के लिये किये गए सबसे महत्त्वपूर्ण उपाय है। स्वतंत्रता के पश्चात भारत में भूमि सुधार कार्यक्रम विभिन्न चरणों में विकसित हुआ है और इन सुधारों के अंतर्गत, बिचौलियों का उन्मूलन, काश्तकारी सुधार, जोत का समेकन, प्रति परिवार जोत का निर्धारण और भूमिहीनों के मध्य अधिशेष भूमि का वितरण करना आदि शामिल है। अतः विकल्प (B) सही है।


3. वैश्विक प्रतियोगित्व रिपोर्ट (ग्लोबल कम्पिटिटिवनेस रिपोर्ट) कौन प्रकाशित करता है?

(a) अंतर्राष्ट्रीय मुद्रा कोष

(b) संयुक्त राष्ट्र व्यापार एवं विकास सम्मेलन (यूनाइटेड नेशंस कॉन्फरेन्स ऑन ट्रेड एंड डेवलपमेंट)

(c) विश्व आर्थिक मंच (वर्ल्ड इकनॉमिक फोरम)

(d) विश्व बैंक

उत्तर: (c)

व्याख्या: वैश्विक प्रतियोगित्व रिपोर्ट (ग्लोबल कम्पिटिटिवनेस रिपोर्ट) विश्व आर्थिक मंच द्वारा प्रकाशित एक वार्षिक रिपोर्ट है।

source:

https://www.drishtiias.com/hindi/daily-updates/daily-news-analysis/prelims-fact-23-04-2018


4. ‘चार्टर एक्ट, 1813’ के संबंध में, निम्नलिखित कथनों पर विचार कीजिये:

1. इसने भारत में ईस्ट इंडिया कंपनी के व्यापार एकाधिपत्य को, चाय का व्यापार तथा चीन के साथ व्यापार को छोड़कर, समाप्त कर दिया।
2. इसने कंपनी द्वारा अधिकार में लिये गए भारतीय राज्यक्षेत्रों पर ब्रिटिश राज (क्राउन) की संप्रभुता को सुदृढ़ कर दिया।
3. भारत का राजस्व अब ब्रिटिश संसद के नियंत्रण में आ गया था।

उपर्युक्त में से कौन-से कथन सही हैं?

(a) केवल 1 और 2   

(b) केवल 2 और 3

(c) केवल 1 और 3   

(d) 1, 2 और 3

उत्तर: (a)

व्याख्या: चार्टर अधिनियम 1813 ने चीन से व्यापार और चाय के व्यापार को छोड़कर भारत में ईस्ट इंडिया कंपनी के व्यापार एकाधिकार को समाप्त कर दिया। इसके अलावा इसने कंपनी को और अगले 20 वर्षों के लिये भारतीय प्रदेशों तथा राजस्व पर नियंत्रण का अधिकार दे दिया किंतु इसने यह भी स्पष्ट किया इससे इन प्रदेशों का ब्रिटिश क्राउन की संप्रभुता पर कोई प्रभाव नही पड़ेगा।

Source: Spectrum modern India page 566-567, BL Grover page 370


5. स्वदेशी आंदोलन के संदर्भ में, निम्नलिखित कथनों पर विचार कीजिये:

1. इसने देशी शिल्पकारों के कौशल तथा उद्योगों को पुनर्जीवित करने में योगदान किया।
2. स्वदेशी आंदोलन के एक अवयव के रूप में राष्ट्रीय शिक्षा परिषद् की स्थापना हुई थी।

उपर्युक्त में से कौन-सा/से कथन सही है/हैं?

(a) केवल 1       

(b) केवल 2

(c) 1 और 2 दोनों

(d) न तो 1, न ही 2

उत्तर: (c)

व्याख्या: स्वदेशी आंदोलन की शुरुआत वर्ष 1905 में बंगाल के विभाजन के विरोध में हुई थी। इसने स्वदेशी के माध्यम से आत्मनिर्भरता पर ज़ोर दिया एवं स्वदेशी कारीगरों, शिल्प और उद्योगों के पुनरुद्धार में महत्त्वपूर्ण योगदान दिया। अगस्त 1906 में स्वदेशी आंदोलन के एक भाग के रूप में राष्ट्रीय शिक्षा के कार्यक्रम को गति प्रदान करने के लिये राष्ट्रीय शिक्षा परिषद की स्थापना की गई थी।

Source: Bipan Chandra page 130-131


6. निम्नलिखित युग्मों पर विचार कीजिये:

  आंदोलन/संगठन                               नायक (लीडर)

1. अखिल भारतीय अस्पृश्यता विरोधी लीग    : महात्मा गांधी
2. अखिल भारतीय किसान सभा               : स्वामी सहजानंद सरस्वती
3. आत्मसम्मान आंदोलन                       : ई.वी. रामास्वामी नायकर

उपर्युक्त में से कौन-सा/से युग्म सही सुमेलित है/हैं?

(a) केवल 1         

(b) केवल 1 और 2

(c) केवल 2 और 3

(d) 1, 2 और 3

उत्तर: (d)

व्याख्या: महात्मा गांधी ने वर्ष 1932 में अखिल भारतीय अस्पृश्यता विरोधी लीग की स्थापना की जो बाद में हरिजन सेवक संघ के रूप में स्थापित हुई। अखिल भारतीय किसान सभा की स्थापना वर्ष 1936 में स्वामी सहजानंद सरस्वती ने औपनिवेशिक भारत में किसान आंदोलनों का नेतृत्त्व करने के लिये की थी। ई. वी. रामास्वामी नायकर (पेरियार) ने वर्ष 1925 में अस्पृश्यता के खिलाफ आत्मसम्मान आंदोलन की शुरुआत की थी।

Source: Spectrum modern India pg 438, Bipan Chandra pg 344, Ncert 8 page 119


7. निम्नलिखित में से कौन-सा एक हड़प्पा स्थल नहीं है?

(a) चन्हुदड़ो   

(b) कोट दीजी

(c) सोहगौरा   

(d) देसलपुर

उत्तर: (c)

व्याख्या: कोट दीजी (वर्तमान में पाकिस्तान में अवस्थित) एक प्रारंभिक हड़प्पा पुरास्थल था। चन्हुदड़ो (पाकिस्तान) और देसलपुर (गुजरात) विकसित हड़प्पा पुरास्थल हैं।

source:

https://www.drishtiias.com/hindi/quiz/result/845#answer-listing


8. निम्नलिखित में से किस उभारदार मूर्तिशिल्प (रिलीफ स्कल्प्चर) शिलालेख में अशोक के प्रस्तर रूपचित्र के साथ ‘राण्यो अशोक’ (राजा अशोक) उल्लिखित है?

(a) कंगनहल्ली   

(b) साँची

(c) शाहबाजगढ़ी 

(d) सोहगौरा

उत्तर: (a)

व्याख्या: कर्नाटक में कंगनहल्ली ब्राह्मी लिपि में उत्कीर्णित एक शिलालेख है जिसमें राण्यो अशोक अर्थात् राजा अशोक का एक मूर्त चित्रण मिलता है।

Source: Upinder Singh page 333


9. निम्नलिखित पर विचार कीजिये:

1. बुद्ध में देवत्वारोपण
2. बोधिसत्त्व के पथ पर चलना
3. मूर्ति उपासना तथा अनुष्ठान

उपर्युक्त में से कौन-सी विशेषता/विशेषताएँ महायान बौद्धमत की है/हैं?

(a) केवल 1         

(b) केवल 1 और 2

(c) केवल 2 और 3

(d) 1, 2 और 3

उत्तर: (d)

व्याख्या: बौद्ध धर्म की महायान शाखा ने भक्ति, प्रतीक पूजा आदि के माध्यम से बुद्ध में देवत्वारोपण को बढ़ावा दिया तथा मूर्ति उपासना और अनुष्ठानों के माध्यम से बोधिसत्त्व के पथ पर चलने का मार्ग प्रशस्त किया।

source:

https://www.drishtiias.com/hindi/quiz/result/834#answer-listing


10. गुप्त काल के दौरान भारत में बलात् श्रम (विष्टि) के संदर्भ में, निम्नलिखित में से कौन-सा कथन सही है?

(a) इसे राज्य के लिये आय का एक स्रोत, जनता द्वारा दिया जाने वाला एक प्रकार का कर, माना जाता था।

(b) यह गुप्त साम्राज्य के मध्य प्रदेश तथा काठियावाड़ क्षेत्रों में पूर्णत: अविद्यमान था।

(c) बलात् श्रमिक साप्ताहिक मज़दूरी का हकदार होता था।

(d) मज़दूर के ज्येष्ठ पुत्र को बलात् श्रमिक के रूप में भेज दिया जाता था।

उत्तर: (a)

व्याख्या: विष्टि को राज्य के लिये आय का एक स्रोत माना जाता था, जो प्रजा द्वारा राज्य को दिये जाने वाला एक प्रकार का कर था।

Source: Upinder Singh page 508


11. किसके राज्य में ‘कल्याण मंडप’ की रचना मंदिर-निर्माण का एक विशिष्ट अभिलक्षण था?

(a) चालुक्य     

(b) चंदेल

(c) राष्ट्रकूट     

(d) विजयनगर

उत्तर: (d)

व्याख्या: कल्याण मंडप विजयनगर साम्राज्य में हम्पी के विट्ठल मंदिर में दैवीय विवाहों के आयोजन में प्रयुक्त होने वाला एक कल्याण मंडप था।

source:

https://www.drishtiias.com/hindi/mains-practice-question/question-537


12. निम्नलिखित कथनों पर विचार कीजिये:

1. दिल्ली सल्तनत के राजस्व प्रशासन में राजस्व वसूली के प्रभारी को ‘आमिल’ कहा जाता था।
2. दिल्ली के सुल्तानों का इक्ता प्रणाली एक प्राचीन देशी संस्था थी।
3. ‘मीर बख्शी’ का पद दिल्ली के खलजी सुल्तानों के शासनकाल में अस्तित्व में आया।

उपर्युक्त में से कौन-सा/से कथन सही है/हैं?

(a) केवल 1   

(b) केवल 1 और 2

(c) केवल 3     

(d) 1, 2 और 3

उत्तर: (a)

व्याख्या: दिल्ली सल्तनत में किसानों से प्रत्यक्ष कर वसूली और ज़मीन की पैमाइश का कार्य आमिलों के ज़िम्मे था। इक्ता प्रणाली पश्चिम एशिया में मुख्यतः फारस में विकसित हुई थी। मीर बख्शी मुगल भारत में सैन्य विभाग का प्रमुख था, जबकि दिल्ली सल्तनत में, दीवान-ए-अर्ज़ (बलबन द्वारा स्थापित) के अंतर्गत आरिज़ विभाग में महत्त्वपूर्ण सुधार अलाउद्दीन खिलज़ी द्वारा किये गए थे इसने दाग प्रणाली की शुरुआत की थी जिसका उद्देश्य घोड़ों की गुणवत्ता में सुधार करने के साथ-साथ इनकी भ्रामक गणना पर अंकुश लगाना था।

Source: Satish Chandra History of Medieval India (800-1700) pg 74, Medieval India old Ncert pg 96, 201, 83


13. निम्नलिखित कथनों पर विचार कीजिये:

1. संत निम्बार्क, अकबर के समकालीन थे।
2. संत कबीर, शेख अहमद सरहिंदी से अत्यधिक प्रभावित थे।

उपर्युक्त में से कौन-सा/से कथन सही है/हैं?

(a) केवल 1       

(b) केवल 2

(c) 1 और 2 दोनों 

(d) न तो 1, न ही 2

उत्तर: (d)

व्याख्या: संत निम्बार्क 13वीं सदी के महान दार्शनिक थे जबकि संत कबीर वर्ष 1556-1605 अकबर के समकालीन 16वीं सदी के कवि थे। एवं शेख अहमद सरहिंदी 17वीं सदी के महत्त्वपूर्ण कवि थे।

Source: Ncert 12th pg 167


14. भारत में ब्रिटिश औपनिवेशिक शासन के संदर्भ में, निम्नलिखित कथनों पर विचार कीजिये:

1. महात्मा गांधी ‘गिरमिटिया (इंडेंचर्ड लेबर)’ प्रणाली के उन्मूलन में सहायक थे।
2. लॉर्ड चेम्सफोर्ड की ‘वॉर कॉन्फरेन्स’ में महात्मा गांधी ने विश्व युद्ध के लिये भारतीयों की भरती से संबंधित प्रस्ताव का समर्थन नहीं किया था।
3. भारत के लोगों द्वारा नमक कानून तोड़े जाने के परिणामस्वरूप, औपनिवेशिक शासकों द्वारा भारतीय राष्ट्रीय कॉन्ग्रेस को अवैध घोषित कर दिया गया था।

उपर्युक्त में से कौन-से कथन सही हैं?

(a) केवल 1 और 2

(b) केवल 1 और 3

(c) केवल 2 और 3

(d) 1, 2 और 3

उत्तर: (b)

व्याख्या: दक्षिण अफ्रीका में, ट्रांसवाल की सरकार के एशियाई कानून संशोधन अधिनियम के अंतर्गत भारतीय आबादी के अनिवार्य पंजीकरण के विरोध में गांधीजी ने अहिंसक विरोध का आवाहन किया था, जिसके परिणामस्वरूप भारतीय राहत अधिनियम 1914 पारित किया गया - जिसने ऐसे भारतीय नागरिकों, जिन्होंने अपने अनुबंधों का नवीनीकरण नहीं किया था, पर अतिरिक्त कर को समाप्त कर दिया और भारतीय विवाहों की वैधता को मान्यता प्रदान की थी।

27 नवंबर, 1920 के यंग इंडिया लेख - "टू एव्री इंग्लिशमेन इन इंडिया" में गांधी ने अपने भर्ती अभियान का उल्लेख किया है, जो दिल्ली के युद्ध सम्मेलन में किये गए वादे के अनुसार प्रथम विश्व युद्ध में ब्रिटिश भागीदारी के समर्थन में कैरा ज़िले (गुजरात) में चलाया गया था।

जैसे ही वर्ष 1930 के अप्रैल महीने में नमक यात्रा का समापन हुआ, जून और अगस्त के मध्य कांग्रेस कार्य समितियों के कई नेताओं को गिरफ्तार कर लिया गया तथा राजनीतिक के रूप में कांग्रेस को ब्रिटिश प्रशासन द्वारा अवैध घोषित कर दिया गया था।

Source:

BipanChandra-pg-279) http://www.bb`c.com/travel/story/20190325-the-birthplace-of-gandhis-peaceful-protest


15. भारतीय राष्ट्रीय आंदोलन के संदर्भ में, निम्नलिखित युग्मों पर विचार कीजिये:

    व्यक्ति                  धारित पद

1. सर तेज बहादुर सप्रू  : अध्यक्ष, अखिल भारतीय उदार संघ
2. के.सी. नियोगी        : सदस्य, संविधान सभा
3. पी.सी. जोशी          : महासचिव, भारतीय साम्यवादी दल

उपर्युक्त में से कौन-सा/से युग्म सही सुमेलित है/हैं?

(a) केवल 1   

(b) केवल 1 और 2

(c) केवल 3   

(d) 1, 2 और 3

उत्तर: (d)

व्याख्या: तेज बहादुर सप्रू लिबरल पार्टी ऑफ इंडिया के एक प्रमुख सदस्य और अखिल भारतीय उदार संघ के अध्यक्ष थे। के.सी. नियोगी संविधान सभा के सदस्य और प्रथम वित्त आयोग के अध्यक्ष थे। जबकि पी.सी. जोशी भारतीय साम्यवादी दल के महासचिव थे।


16. मियाँ तानसेन के संदर्भ में, निम्नलिखित में से कौन-सा कथन सही नहीं है?

(a) सम्राट अकबर द्वारा इन्हें दी गई उपाधि तानसेन थी।

(b) तानसेन ने हिन्दू देवी-देवताओं से संबंधित ध्रुपदों की रचना की।

(c) तानसेन ने अपने संरक्षकों से संबंधित गानों की रचना की।

(d) तानसेन ने अनेक रागों की मौलिक रचना की।

उत्तर: (a)

व्याख्या: तानसेन की उपाधि ग्वालियर के राजा विक्रमजीत ने दी थी। तानसेन बांधवगढ़ (रीवा) के राजा रामचंद्र के दरबार में दरबारी संगीतकार थे।

Source: https://www.thehindu.com/entertainment/music/the-legend-of-mian-tansen/article22893454.ece


17. इनमें से किस मुगल सम्राट ने सचित्र पांडुलिपियों से ध्यान हटाकर चित्राधार (एलबम) और वैयक्तिक रूपचित्रों पर अधिक ज़ोर दिया?

(a) हुमायूँ   

(b) अकबर

(c) जहाँगीर 

(d) शाहजहाँ

उत्तर: (c)

व्याख्या: अकबर के शासनकाल के दौरान कई पांडुलिपियों का चित्रण किया गया था, जिनमें महाभारत का फारसी अनुवाद रज़्मनामा प्रमुख है। जहाँगीर के शासनकाल के दौरान पांडुलिपियों का महत्त्व कम हो गया था और व्यक्तिगत चित्रण अधिक प्रमुख हो गए थे।

Source: Pearson Indian history Manual, NCERT 12th Kings and Chronicles


18. निम्नलिखित में से कौन-सा नैशनल पार्क पूर्णतया शीतोष्ण अल्पाइन कटिबंध में स्थित है?

(a) मानस नैशनल पार्क       

(b) नामदफा नैशनल पार्क

(c) नेओरा घाटी नैशनल पार्क

(d) पूलाें की घाटी नैशनल पार्क

उत्तर: (d)

व्याख्या: उत्तराखंड के चमोली ज़िले में स्थित फूलों की घाटी राष्ट्रीय उद्यान और स्थानिक अल्पाइन फूलों के चारागाह स्थलों एवं वनस्पतियों की विविधता के लिये जाना जाता है। समुद्र तल से लगभग 3255 मीटर की ऊँचाई पर स्थित यह महत्त्वपूर्ण उद्यान लगभग 87.5 वर्ग किमी भूमि में फैला हुआ है।

घाटी में तीन प्रकार के उप-अल्पाइन क्षेत्र हैं: वृक्षों की शीर्ष ऊँचाई की सीमा के आधार पर,  3200 मीटर से 3500 मीटर के मध्य तलहटी अल्पाइन , 3500 मीटर से 3700 मीटर के मध्य निचले अल्पाइन और 3700 मीटर से ऊपर उच्च अल्पाइन क्षेत्र हैं।

क्रमश: उत्तर और दक्षिण में ज़ांस्कर और महान हिमालय पर्वतमाला के मध्य तथा पूर्वी और पश्चिमी हिमालय के बीच एक संक्रमण क्षेत्र के मध्य अवस्थिति के कारण यहाँ वनस्पतिय प्रजातियों की समृद्ध विविधता देखने को मिलती है।

source:

https://www.drishtiias.com/hindi/quiz/quizlist/60-steps-to-prelims


19. अटल नवप्रवर्तन (इनोवेशन) मिशन किसके अधीन स्थापित किया गया है?

(a) विज्ञान एवं प्रौद्योगिकी विभाग

(b) श्रम एवं रोजगार मंत्रालय

(c) नीति (NITI) आयोग

(d) कौशल विकास एवं उद्यमिता मंत्रालय

उत्तर: (c)

व्याख्या: अटल नवप्रवर्तन मिशन का उद्देश्य देश भर में विद्यालयों, विश्वविद्यालयों, अनुसंधान संस्थानों, MSME और उद्योग स्तरों पर नवाचार और उद्यमिता का एक पारिस्थितिकी तंत्र निर्मित करना तथा उसे प्रोत्साहित करना है। वर्ष 2015 के बजट भाषण में माननीय वित्त मंत्री की घोषणा के अनुसार नीति आयोग के अधीन मिशन की शुरुआत की गई है।

source:

https://www.drishtiias.com/hindi/summary-of-important-reports/niti-aayog-policy-report-2017-18

https://www.drishtiias.com/hindi/daily-updates/daily-news-analysis/budget-2018-enhances-niti-role-in-the-national-policy-frontier-technologies


20. जून की 21वीं तारीख को सूर्य

(a) उत्तरध्रुवीय वृत्त पर क्षितिज के नीचे नहीं डूबता है

(b) दक्षिणध्रुवीय वृत्त पर क्षितिज के नीचे नहीं डूबता है

(c) मध्याह्न में भूमध्यरेखा पर ऊर्ध्वाधर रूप से व्योमस्थ चमकता है

(d) मकर-रेखा पर ऊर्ध्वाधर रूप से व्योमस्थ चमकता है

उत्तर: (a)

व्याख्या: 21 जून को 'ग्रीष्म संक्रांति' के दौरान, उत्तरी गोलार्द्ध में वर्ष का सबसे लंबा दिन होता है, जबकि दक्षिणी गोलार्द्ध में सबसे छोटा दिन होता है। इस समय, पृथ्वी का उत्तरी ध्रुव सूर्य की ओर अपने अधिकतम झुकाव पर होता है और सूर्य 23.5º उत्तरी अक्षांश पर, यानी कर्क रेखा के साथ सीधे ऊपर की ओर दिखाई देता है। 

सूर्य ग्रीष्म संक्रांति के दौरान उत्तरध्रुवीय वृत्त पर क्षितिज के नीचे नहीं डूबता है, क्योंकि यहाँ सूर्योदय और सूर्यास्त उत्तरी आकाश में एकाग्र होने लगते हैं। इसके विपरीत 22 दिसंबर को शीतकालीन संक्रांति के दौरान अंटार्कटिक वृत्त में भी यही घटना होती है। अतः विकल्प (A) सही है।

source:

https://www.drishtiias.com/hindi/daily-updates/daily-news-analysis/the-tilt-or-why-thursday-had-longer-daylight-hours-than-any-other-day


21. निम्नलिखित में से कौन-सा एक पादप-समूह ‘नवीन विश्व (न्यू वर्ल्ड)’ में कृषि-योग्य बनाया गया तथा इसका ‘प्राचीन विश्व (ओल्ड वर्ल्ड)’ में प्रचलन शुरू किया गया?

(a) तंबाकू, कोको और रबड़

(b) तंबाकू, कपास और रबड़

(c) कपास, कॉफी और गन्ना

(d) रबड़, कॉफी और गेहूँ

उत्तर: (a)

व्याख्या: "नवीन विश्व" अमेरिकी महाद्वीप को संदर्भित करता है जिसे स्पेन, पुर्तगाली और फ्राँसीसी आदि यूरोपीय शक्तियों द्वारा खोजा गया था; तंबाकू, कोको और रबड़ अमेरिका की मूल फसलें थीं और इनका प्रचलन प्राचीन विश्व (एशिया) में किया गया था।


22. निम्नलिखित कथनों पर विचार कीजिये:

1. एशियाई शेर प्राकृतिक रूप से सिर्प भारत में पाया जाता है।
2. दो-कूबड़ वाला ऊँट प्राकृतिक रूप से सिर्प भारत में पाया जाता है।
3. एक-सींग वाला गैंडा प्राकृतिक रूप से सिर्प भारत में पाया जाता है।

उपर्युक्त में से कौन-सा/से कथन सही है/हैं?

(a) केवल 1         

(b) केवल 2

(c) केवल 1 और 3

(d) 1, 2 और 3

उत्तर: (a)

व्याख्या: एशियाई शेर जो कभी फारस (ईरान) से लेकर पलामू पूर्वी भारत तक फैले हुए थे, अत्यधिक शिकार और इनके अधिवास को हुए नुकसान से लगभग विलुप्त हो गए थे। 1890 के दशक के अंत तक गुजरात के गिर के जंगलों में 50 से भी कम शेरों की आबादी बची थी। राज्य सरकार और केंद्र सरकार द्वारा समय पर कड़ी सुरक्षा प्रदान कर किये गए संरक्षण प्रयासों के परिणामस्वरूप वर्तमान में एशियाई शेरों की संख्या 500 से अधिक हो गई है। अतः कथन 1 सही है। दोहरे कूबड़ वाला ऊँट गोबी रेगिस्तान का मूल अधिवासी है और यह मंगोलिया, चीन, कज़ाकिस्तान, तुर्कमेनिस्तान, उज़्बेकिस्तान, तथा अफगानिस्तान के ठंडे रेगिस्तानी क्षेत्रों में पाया जाता है। अतः कथन 2 सही नहीं है। पूर्वोत्तर भारत और नेपाल के तराई घासस्थलों में एक सींग वाले गैंडों की आबादी में लगभग 3,500 गैंडों की वृद्धि हुई है। अतः कथन 3 सही नहीं है।

source:

https://www.drishtiias.com/hindi/daily-updates/prelims-facts/prelims-facts-12-03-2019

https://www.drishtiias.com/hindi/daily-updates/daily-news-analysis/asiatic-lion-conservation-project-1-lion

https://www.drishtiias.com/hindi/daily-updates/daily-news-analysis/rhinos-without-borders-is-conservation-credo


23. निम्नलिखित युग्मों पर विचार कीजिये:

   प्रसिद्ध स्थान       नदी

1. पंढरपुर           : चंद्रभागा
2. तिरुचिरापल्ली    : कावेरी
3. हंपी               : मालप्रभा

उपर्युक्त में से कौन-से युग्म सही सुमेलित हैं?

(a) केवल 1 और 2 

(b) केवल 2 और 3

(c) केवल 1 और 3 

(d) 1, 2 और 3

उत्तर: (a)

व्याख्या: पंढरपुर में श्री विठ्ठल एवं श्री रुक्मिणी का पवित्र स्थान है। इसे दक्षिण भारत की काशी और महाराष्ट्र राज्य के कुलदैवत के नाम से भी जाना जाता है। चंद्रभागा (भीमा) नदी पंढरपुर नगर से होकर प्रवाहित होती है। कावेरी नदी के किनारे स्थित तिरुचिरापल्ली तमिलनाडु का चौथा सबसे बड़ा नगर है। यह प्रारंभिक चोलों का दुर्ग हुआ करता था जो कि बाद में पल्लवों के अधीन हो गया।

हंपी कर्नाटक राज्य में तुंगभद्रा नदी के दक्षिणी किनारे अवस्थित महत्त्वपूर्ण ऐतिहासिक स्थल है जिसे भारत में यूनेस्को विश्व धरोहर स्थल का दर्ज़ा प्राप्त है।

source:

https://www.drishtiias.com/hindi/quiz/result/1195#answer-listing


24. किसी दिये गए वर्ष में भारत में कुछ राज्यों में आधिकारिक गरीबी रेखाएँ अन्य राज्यों की तुलना में उच्चतर हैं, क्योंकि

(a) गरीबी की दर अलग-अलग राज्य में अलग-अलग होती है

(b) कीमत-स्तर अलग-अलग राज्य में अलग-अलग होता है

(c) सकल राज्य उत्पाद अलग-अलग राज्य में अलग-अलग होता है

(d) सार्वजनिक वितरण की गुणता अलग-अलग राज्य में अलग-अलग होती है

उत्तर: (b)

व्याख्या: कीमत-स्तर अलग-अलग राज्य में भिन्न-भिन्न होता है जिसके कारण गरीबी रेखाएँ अलग-अलग राज्यों में भिन्न-भिन्न होती हैं।


25. निम्नलिखित में से किसके संदर्भ में, कुछ वैज्ञानिक पक्षाभ मेघ विरलन तकनीक तथा समतापमंडल में सल्पेट वायुविलय अंत:क्षेपण के उपयोग का सुझाव देते हैं?

(a) कुछ क्षेत्रों में कृत्रिम वर्षा करवाने के लिये

(b) उष्णकटिबंधीय चक्रवातों की बारंबारता और तीव्रता को कम करने के लिये

(c) पृथ्वी पर सौर पवनों के प्रतिकूल प्रभाव को कम करने के लिये

(d) भूमंडलीय तापन को कम करने के लिये

उत्तर: (d)

व्याख्या: भूमंडलीय तापन की पूर्व स्थिति प्राप्त करने के लिये जियो-इंजीनियरिंग दृष्टिकोण - 'पक्षाभ मेघ विरलन तकनीक तथा समतापमंडल में सल्पेट वायुविलय अंत:क्षेपण के माध्यम से पक्षाभ मेघों के दीर्घतरंगीय विकरण प्रभावों को संशोधित करके भूमंडलीय तापन को संभावित रूप से कम किया जा सकता है।'

source:

https://www.drishtiias.com/hindi/daily-updates/daily-news-analysis/worried-over-weak-rain-forecast-karnataka-plans-cloud-seeding


26. निम्नलिखित में से किसके संदर्भ में, ‘ताप-अपघटन और प्लाज़्मा गैसीकरण’ शब्दों का उल्लेख किया गया है?

(a) दुर्लभ (रेअर) भू-तत्त्वों का निष्कर्षण

(b) प्राकृतिक गैस निष्कर्षण प्रौद्योगिकी

(c) हाइड्रोजन ईंधन-आधारित ऑटोमोबाइल

(d) अपशिष्ट-से-ऊर्जा प्रौद्योगिकी

उत्तर: (d)

व्याख्या: तीन वर्षीय कार्य एजेंडा (वर्ष 2017-18 से 2019-20 तक) विकसित करने में, नीति आयोग ने नगरपालिका ठोस अपशिष्ट (MSW) के मुद्दे को संबोधित करने हेतु एक व्यापक रूपरेखा तैयार की है। कार्य एजेंडा द्वारा सुझाया गया समाधान दो विधियों को; बड़ी नगरपालिकाओं के लिये अपशिष्ट-से-ऊर्जा भस्मीकरण विधि को तथा छोटे शहरों एवं अर्ध-शहरी क्षेत्रों के लिये अपशिष्ट निपटान की कंपोस्टिंग विधि को शामिल करता है। समाधान के रूप में भस्मीकरण विधि का प्रस्ताव करते हुए नीति आयोग ने थर्मल पाइरोलिसिस और प्लाज़्मा गैसीकरण प्रौद्योगिकियों के लाभ-लागत अनुपात का भी आकलन किया है।

source:

https://www.drishtiias.com/hindi/mains/mains-strategy/mains-exam-paper-3-strategy-page


27. निम्नलिखित में से कौन-से अगस्त्यमाला जीवमंडल रिज़र्व में आते हैं?

(a) नेबयार, पेप्पारा और शेंदुर्ने वन्य प्राणी अभयारण्य; और कलाकड़ मुंदन्थुराई बाघ रिज़र्व

(b) मुदुमलाई सत्यमंगलम और वायनाड वन्य प्राणी अभयारण्य; और साइलेंट वैली नैशनल पार्क

(c) कैंडिन्य, गुंडला ब्रह्मेश्वरम और पापीकोंडा वन्य प्राणी अभयारण्य; और मुकुर्थी नैशनल पार्क

(d) कावल और श्रीवेंकटेश्वर वन्य प्राणी अभयारण्य; और नागार्जुनसागर-श्रीशैलम बाघ रिज़र्व

उत्तर: (a)

व्याख्या: अगस्त्यमाला जीवमंडल भारत के दक्षिण में पश्चिमी घाट में अवस्थित एक जीवमंडल है। यहाँ मुख्यतः उष्णकटिबंधीय वन पाए जाते हैं एवं यहाँ वनस्पतियों की लगभग 400 स्थानिक प्रजातियों साथ-साथ संवहनी पौधों की 2,254 प्रजातियाँ पाई जाती हैं। यहाँ कृषि योग्य पादपों, विशेष रूप से इलायची, जामुन, ज़ायफल, काली मिर्च और केला का एक अनूठा भंडार देखने को मिलता है। साथ ही यहाँ तीन वन्य प्राणी अभयारण्य, शेंदुर्ने, पेप्पारा और नेय्यार एंव  कलाकड़ मुंदन्थुराई बाघ रिज़र्व भी स्थित है।


28. निम्नलिखित कथनों पर विचार कीजिये:

1. समुद्री कच्छपों की कुछ जातियाँ शाकभक्षी होती हैं।
2. मछली की कुछ जातियाँ शाकभक्षी होती हैं।
3. समुद्री स्तनपायियों की कुछ जातियाँ शाकभक्षी होती हैं।
4. साँपों की कुछ जातियाँ सजीवप्रजक होती हैं।

उपर्युक्त में से कौन-से कथन सही हैं?

(a) केवल 1 और 3

(b) केवल 2, 3 और 4

(c) केवल 2 और 4

(d) 1, 2, 3 और 4

उत्तर: (d)

व्याख्या: हरे समुद्री कछुए समुद्री घास और शैवाल से अपना भोजन प्राप्त करते हैं, हालाँकि किशोर हरे कछुए, केकड़ों, स्पंज और जेलिफिश के माध्यम से अपना भोजन प्राप्त करते हैं। तोता मछली शाकभक्षी  होती है यह चट्टान से मूंगे के छोटे-छोटे टुकड़ों को हटाकर शैवाल प्राप्त करती है। मछली के चवाने वाले दाँत, उसके गले में स्थित होते हैं, जो मूंगा को महीन कर देते हैं। दाँत मछली को पॉलीप के अंदर से शैवाल निकालने में सहायक होते हैं। मैनाटी, जिसे समुद्री गाय भी कहा जाता है, स्तनधारियों की एक प्रज़ाति है जो उथले तटीय क्षेत्रों के उष्मीय जल में रहते हैं और शाकभक्षी होते हैं एवं समुद्री वनस्पति से अपना भोजन प्राप्त करते हैं। सजीवप्रजक सर्प अपरा और जर्दी थैली के माध्यम से अपने बच्चों का पोषण करते हैं, जो सरीसृपों के मध्य अत्यधिक असामान्य लक्षण है।

Source: https://sciencing.com/list-herbivores-ocean-8599405.html


29. निम्नलिखित युग्मों पर विचार कीजिये:

   वन्य प्राणी                   प्राकृतिक रूप से कहाँ पाए जाते हैं

1. नीले मीनपक्ष वाली महाशीर           : कावेरी नदी
2. इरावदी डॉल्फिन                       : चंबल नदी
3. मोरचाभ (रस्टी)-चित्तीदार बिल्ली     : पूर्वी घाट

उपर्युक्त में से कौन-से युग्म सही सुमेलित हैं?

(a) केवल 1 और 2   

(b) केवल 2 और 3

(c) केवल 1 और 3   

(d) 1, 2 और 3

उत्तर: (c)

व्याख्या: पश्चिमी घाट क्षेत्र अलवणीय जल की जैव विविधता के लिये प्रसिद्ध है। महाशीर,जो कि टोर वंश से अपना संबंध रखती है, इसे बहुमूल्य सांस्कृतिक क्रीडा मत्स्य के रूप में भी जाना जाता है। कावेरी नदी में महाशीर की दो प्रजातियाँ शामिल हैं- एक "नीले मीनपक्ष वाली महाशीर " और एक "नारंगी मीनपक्ष एवं कूबड़ युक्त महाशीर। अतः युग्म 1 सही सुमेलित है। इरावदी डॉल्फिन उथले, निकट-तटीय उष्णकटिबंधीय और उपोष्णकटिबंधीय समुद्री जल में पाई जाती हैं। यह मुख्यतः समुद्री मुहानों और अर्ध-संलग्न जल क्षत्रों जैसे कि खाड़ी आदि में पायी जाती हैं जो कि आमतौर पर मैंग्रोव वनों के निकट अवस्थित होती हैं। अतः युग्म 2 सही सुमेलित नहीं है। पूर्वी घाट के वन कुछ लुप्तप्राय और कम ज्ञात स्तनपायी प्रजातियों जैसे फिशिंग कैट, रस्टी स्पॉटेड कैट, भारतीय पैंगोलिन आदि का अधिवास भी है। अतः युग्म 3 सही सुमेलित है।

Source: https://www.downtoearth.org.in/news/iconic-indian-fish-on-the-verge-of-extinction-says-study-49802

http://wwf.panda.org/knowledge_hub/endangered_species/cetaceans/about/irrawaddy_dolphin/

https://www.thehindu.com/news/national/andhra-pradesh/road-ecology-the-road-less-travelled/article24585647.ece


30. पर्यावरण में निर्मुक्त हो जाने वाली ‘सूक्ष्ममणिकाओं (माइक्रोबीड्स)’ के विषय में अत्यधिक चिंता क्यों है?

(a) ये समुद्री पारितंत्रों के लिये हानिकारक मानी जाती हैं।

(b) ये बच्चों में त्वचा कैंसर होने का कारण मानी जाती हैं।

(c) ये इतनी छोटी होती हैं कि सिंचित क्षेत्रों में फसल पादपों द्वारा अवशोषित हो जाती हैं।

(d) अक्सर इनका इस्तेमाल खाद्य-पदार्थों में मिलावट के लिये किया जाता है।

उत्तर: (a)

व्याख्या: सूक्ष्ममणिकाएँ (माइक्रोबीड्स) छोटे, ठोस, प्लास्टिक से निर्मित कण हैं जिनका आकर 5 मिली मीटर से कम होता है और जल में में घुलनशील नहीं होते हैं। यह विभिन्न उत्पादों में पाए जाते हैं जिनकी सूची में सौंदर्य प्रसाधन, व्यक्तिगत देखभाल और सफाई उत्पाद शामिल हैं। इन्हें समुद्री और जलीय पारिस्थितिकी तंत्र के लिये हानिकारक माना जाता है।

Source: https://www.environment.gov.au/protection/waste-resource-recovery/plastics-and-packaging/plastic-microbeads


31. हाल ही में हमारे देश में हिमालयी बिच्छू-बूटी (जिरार्डीनिया डाइवर्सीफोलिया) के महत्त्व के बारे में बढ़ती हुई जागरूकता थी, क्योंकि यह पाया गया है कि

(a) यह प्रति-मलेरिया औषध का संधारणीय स्रोत है

(b) यह जैव डीज़ल का संधारणीय स्रोत है

(c) यह कागज़ उद्योग के लिये लुगदी का संधारणीय स्रोत है

(d) यह वस्त्रतंतु का संधारणीय स्रोत है

उत्तर: (d)

व्याख्या: हिमालयी बिच्छू-बूटी (जिरार्डीनिया डाइवर्सीफोलिया) यह वस्त्रतंतु का संधारणीय स्रोत है। हिमालयी भारतीय बिच्छू-बूटी के लिये भारत सरकार ने जूट और तंतु प्रौद्योगिकी विभाग, जूट प्रौद्योगिकी संस्थान, कोलकाता विश्वविद्यालय को हिमालयी भारतीय बिच्छू-बूटी से निर्मित उत्पादों के विकास नामक परियोजना को मंजूरी दी है।

Source: http://pib.nic.in/newsite/PrintRelease.aspx?relid=159342


32. निम्नलिखित में से किसका/किनका मापन/आकलन करने के लिये उपग्रह चित्रों/सुदूर संवेदी आँकड़ों का इस्तेमाल किया जाता है?

1. किसी विशेष स्थान की वनस्पति में पर्णहरित का अंश
2. किसी विशेष स्थान के धान के खेतों से ग्रीनहाउस गैस का उत्सर्जन
3. किसी विशेष स्थान का भूपृष्ठ तापमान

नीचे दिये गए कूट का प्रयोग कर सही उत्तर चुनिये:

(a) केवल 1   

(b) केवल 2 और 3

(c) केवल 3   

(d) 1, 2 और 3

उत्तर: (d)

व्याख्या: कथन 1: यूरोपीय अंतरिक्ष एजेंसी के प्रहरी उपग्रह अर्थात् सेंटिनल उपग्रह (Sentinel satellite) में कुछ विशिष्ट वर्णक्रमीय विशेषताएँ हैं जिनका उपयोग किसी विशिष्ट स्थान की वनस्पति में पर्णहरित को मापने के लिये भी किया जा सकता है। अतः कथन 1 सही है।

कथन 2: सुदूर संवेदन के साथ भूपृष्ठ गतिकी निरूपण के आधार पर मेकांग डेल्टा में धान के खेतों से मेथैन उत्सर्जन का अनुमान लगाया जा सकता। अतः कथन 2 सही है।

कथन 3: किसी विशेष स्थान का भूपृष्ठ तापमान को उपग्रह चित्रों/सुदूर संवेदन आँकड़ों की मदद से मापा जा सकता है। अतः कथन 3 सही है।

Source: https://www.downtoearth.org.in/interviews/science-and-technology/-remote-sensing-is-being-used-to-predict-crop-growing-seasons--56372

Covered in Drishti: https://www.drishtiias.com/daily-updates/daily-news-analysis/isro-to-launch-state-of-the-art-satellites


33. निम्नलिखित राज्यों पर विचार कीजिये:

1. छत्तीसगढ़
2. मध्य प्रदेश
3. महाराष्ट्र
4. ओडिशा

उपर्युक्त राज्यों के संदर्भ में, राज्य के कुल क्षेत्रफल की तुलना में वन आच्छादन की प्रतिशतता के आधार पर निम्नलिखित में से कौन-सा सही आरोही अनुक्रम है?

(a) 2, 3, 1, 4

(b) 2, 3, 4, 1

(c) 3, 2, 4, 1

(d) 3, 2, 1, 4

उत्तर: (c)

व्याख्या: भारत वन स्थिति रिपोर्ट 2017

राज्य

कुल क्षेत्रफल में वनों का प्रतिशत (अनुमानित आँकड़े)

1.छत्तीसगढ़

41%

2. मध्यप्रदेश

25%

3. महाराष्ट्र

16%

4. ओडिशा

32%

अतः सही आरोही क्रम है:- (3-2-4-1)

source:

https://www.drishtiias.com/hindi/daily-updates/daily-news-analysis/in-uttarakhand-the-dam-will-grow-in-other-states-of-the-jungle


34. ‘मेथैन हाइड्रेट’ के निक्षेपों के बारे में, निम्नलिखित में से कौन-से कथन सही हैं?

1. भूमंडलीय तापन के कारण इन निक्षेपों से मेथैन गैस का निर्मुक्त होना प्रेरित हो सकता है।
2. ‘मेथैन हाइड्रेट’ के विशाल निक्षेप उत्तरध्रुवीय टुंड्रा में तथा समुद्र अधस्तल के नीचे पाए जाते हैं।
3. वायुमंडल के अंदर मेथैन एक या दो दशक के बाद कार्बन डाइऑक्साइड में ऑक्सीकृत हो जाता है।

नीचे दिये गए कूट का प्रयोग कर सही उत्तर चुनिये:

(a) केवल 1 और 2

(b) केवल 2 और 3

(c) केवल 1 और 3

(d) 1, 2 और 3

उत्तर: (d)

व्याख्या: अशांत समुद्र के नीचे मेथैन हाइड्रेट्स के निक्षेप एक बड़े पारिस्थितिक खतरे को उत्पन्न कर सकते है। यहाँ तक कि अगर इन मेथैन हाइड्रेट निक्षेपों का एक छोटा सा हिस्सा भी प्राकृतिक प्रक्रियाओं के माध्यम से विघटित हो जाता है, तो भूमंडलीय तापन के कारण बड़ी मात्रा में मेथैन गैस निर्मुक्त होना प्रेरित हो सकता है। अतः कथन 1 सही है ।

मेथैन हाइड्रेट्स महाद्वीपीय ढलानों के निचले सीमांत क्षेत्रों के साथ निर्मित होते हैं, जहाँ समुद्र अधस्तल अपेक्षाकृत उथले शेल्फ से नीचे रहता है, आमतौर पर समुद्र की सतह से लगभग 150 मीटर नीचे। जलवायु के लिये गैस हाइड्रेट्स की संवेदनशीलता, तापन घटना की अवधि, समुद्र अधस्तल या टुंड्रा सतह के नीचे गैस हाइड्रेट्स गहराई, और गैस हाइड्रेट्स को पृथक करने हेतु तलछट को गर्म करने के लिये आवश्यक तापन की मात्रा पर, निर्भर करती है। अतः कथन 2 सही है।

मेथैन के साथ समस्या यह है कि यह बिना अपने अवशेष छोड़े नष्ट नहीं होती है, भले ही यह कम अवधि के लिये वातावरण में रहती हो जो कि औसतन 10 वर्ष है। मुक्त ऑक्सीजन की उपस्थिति में, कार्बन का एक मेथैन अणु अपने चार हाइड्रोजन परमाणुओं से कार्बन डाइऑक्साइड बनने के लिये अलग हो जाता है। अतः कथन 3 सही है।

Source: https://www.downtoearth.org.in/news/burning-ice-17856

Covered in Drishti: https://www.drishtiias.com/daily-updates/daily-news-analysis/iit-madras-team-produces-gas-hydrates


35. निम्नलिखित पर विचार कीजिये:

1. कार्बन मोनोक्साइड
2. मेथैन
3. ओज़ोन
4. सल्फर डाइऑक्साइड

फसल/जैव मात्रा के अवशेषों के दहन के कारण वायुमंडल में उपर्युक्त में से कौन-से निर्मुक्त होते हैं?

(a) केवल 1 और 2

(b) केवल 2, 3 और 4

(c) केवल 1 और 4

(d) 1, 2, 3 और 4

उत्तर: (d)

व्याख्या: फसल अवशेषों के दहन से कार्बन डाइऑक्साइड, मेथैन, कार्बन मोनोऑक्साइड, नाइट्रस ऑक्साइड, सल्फर डाइऑक्साइड और सूक्ष्म कण उत्सर्जित होते हैं जो मानवीय स्वास्थ्य को प्रभावित करते हैं।

दहन से होने वाले वायु प्रदूषण के लिये पंजाब और हरियाणा दोनों राज्य ज़िम्मेदार हैं। यह राज्य, पूरे भारत में धान के दहन से होने वाले कुल उत्सर्जन में, 48 प्रतिशत का योगदान करते हैं।

नोट: अधिकांश क्षोभमंडलीय ओज़ोन का निर्माण तब होता है जब नाइट्रोजन ऑक्साइड (NOx), कार्बन मोनोऑक्साइड (CO) और वाष्पशील कार्बनिक यौगिक (VOCs), जैसे कि ज़ाइलीन, सूर्य के प्रकाश की उपस्थिति में वातावरण में अभिक्रिया करते हैं।

source:

https://www.drishtiias.com/hindi/daily-updates/daily-news-editorials/fighting-air-pollution-the-green-way


36. निम्नलिखित युग्मों पर विचार कीजिये:

 सागर                  सागर से लगा हुआ देश

1. ऐड्रिऐटिक सागर   : अल्बानिया
2. काला सागर        : क्रोएशिया
3. कैस्पियन सागर    : कज़ाकिसतान
4. भूमध्य सागर       : मोरक्को
5. लाल सागर         : सीरिया

उपर्युक्त में से कौन-से युग्म सही सुमेलित हैं?

(a) केवल 1, 2 और 4

(b) केवल 1, 3 और 4

(c) केवल 2 और 5     

(d) 1, 2, 3, 4 और 5

उत्तर: (b)

व्याख्या: एड्रियाटिक सागर भूमध्य सागर का एक ही एक भाग है जो इटली के पूर्वी तट और बाल्कन प्रायद्वीप के देशों स्लोवेनिया, क्रोएशिया, मोंटेनेग्रो और अल्बानिया के मध्य स्थित है। अतः युग्म 1 सही सुमेलित है।

काला सागर एक अंतर्देशीय सागर है जो सुदूर-दक्षिणपूर्वी यूरोप और एशिया महाद्वीप के सुदूर-पश्चिमी सीमांत क्षेत्रों तथा तुर्की के मध्य अवस्थित है। यह तुर्की, एवं बुल्गारिया, रोमानिया, यूक्रेन, रूस और जॉर्जिया द्वारा सीमाबद्ध है। अतः युग्म 2 सही सुमेलित नहीं है। यह विकल्प a, c और d को बाहर करता है, सही उत्तर को विलोपन विधि के माध्यम से विकल्प B के रूप में छोड़ता है।

कैस्पियन सागर एशिया और यूरोप महाद्वीप द्वारा सीमाबद्ध है। ईरान, तुर्कमेनिस्तान, कज़ाकिस्तान, अज़रबैजान और रूस इसकी सीमा बनाते हैं।

मोरक्को का भूमध्यसागरीय तट उत्तरी अफ्रीकी तट की पश्चिमी सीमा का निर्माण करता है। यहीं जिब्राल्टर जलडमरू भी है जो भूमध्य सागर और अटलांटिक महासागर के मध्य की कड़ी के रूप में कार्य करता है।

लाल सागर की सीमा छह देश (सऊदी अरब, यमन, मिस्र, सूडान, इरिट्रिया और ज़िबूती) निर्मित करते  हैं।

source:

मैगज़ीन मैप

पेज़ no. 62, 64 नवंबर 2018


37. निम्नलिखित में से कौन-सा देश पिछले पाँच वर्षों के दौरान विश्व में चावल का सबसे बड़ा निर्यातक रहा है?

(a) चीन     

(b) भारत

(c) म्याँमार 

(d) वियतनाम

उत्तर: (b)

व्याख्या: इस दशक की शुरुआत से ही भारत विश्व का शीर्ष चावल निर्यातक रहा है। भारत वर्ष 2011-12 में विश्व का सबसे बड़ा चावल निर्यातक बनकर उभरा, भारत ने थाईलैंड को शीर्ष की स्थिति से प्रतिस्थापित किया था।

Source: India Today, The Hindu, Economic Times, Worldatlas.com


38. निम्नलिखित युग्मों पर विचार कीजिये:

  हिमनद         नदी

1. बंदरपूँछ     : यमुना
2. बारा शिग्री   : चेनाब
3. मिलाम       : मंदाकिनी
4. सियाचिन     : नुब्रा
5. जेमू          : मानस

उपर्युक्त में से कौन-से युग्म सही सुमेलित हैं?

(a) केवल 1, 2 और 4

(b) केवल 1, 3 और 4

(c) केवल 2 और 5   

(d) केवल 3 और 5

उत्तर: (a)

व्याख्या: हिमालय के गढ़वाल मंडल में यमुना नदी द्रोणी का एक महत्त्वपूर्ण हिमनद है, जिसे बंदरपूँछ के नाम से जाना जाता है। यह हिमनद बंदरपूँछ पश्चिम, खतलिंग चोटी और बंदरपूँछ चोटी के उत्तरी ढलानों पर 12 किमी तक विस्तृत है, यह हिमनद तीन हिमगह्वर (सर्क) द्वारा निर्मित होता है जो बाद में यमुना नदी में मिलता है। अत: 1 सुमेलित है।

बारा शिग्री हिमाचल प्रदेश की चँद्र घाटी के लाहौल स्पीति क्षेत्र में स्थित सबसे बड़ा हिमनद है। जो कि लगभग 30 किलोमीटर तक विस्तृत है एवं गंगोत्री के बाद हिमालय का दूसरा सबसे विस्तृत हिमनद है। इसका प्रवाह उत्तर की ओर है और यह चिनाब नदी को जल प्रदान करता है। अतः कथन 2 सही है।

उत्तराखंड के पिथौरागढ़ ज़िले के मुनस्यारी में मिलम हिमनद गोरी गंगा नदी का स्रोत है न कि मंदाकिनी नदी का। गोरी गंगा भी काली नदी की एक महत्त्वपूर्ण सहायक नदी है। अतः कथन 3 सही नहीं है।

लगभग 5,400 मीटर (17,700 फीट) की ऊँचाई पर, कश्मीर में अवस्थित सियाचिन हिमनद एक निषिद्ध क्षेत्र है। अत्यधिक कम ऊँचाई पर यह हिमनद का प्रभाव सौम्य है: यह नुब्रा नदी का स्रोत है, जो सिंधु नदी की एक सहायक नदी है, जो कि पाकिस्तान में प्रवाहित होती हुई अरब सागर में  मिलती है। अतः कथन 4 सही है।

ज़ेमू हिमनद सिक्किम में अवस्थित है एवं पूर्वी हिमालय का सबसे बड़ा हिमनद है। यह कंचनजंगा के आधार पर अवस्थित है और मानस नदी के स्रोतों में से एक है न की तीस्ता नदी का । तीस्ता ब्रह्मपुत्र की एक सहायक नदी है। अतः कथन 5 सही नहीं है।

Source: http://www.visituttrakhand.com/bandarpunch-glacier-trek.php


39. भारत में कार्बोफ्यूरेन, मेथिल पैराथियॉन, फोरेट और ट्राइऐजोफॉस के इस्तेमाल को आशंका से देखा जाता है। ये रसायन किस रूप में इस्तेमाल किये जाते हैं?

(a) कृषि में पीड़कनाशी

(b) संसाधित खाद्यों में परिरक्षक

(c) फल-पक्वन कारक

(d) प्रसाधन सामग्री में नमी बनाए रखने वाले कारक

उत्तर: (a)

व्याख्या: कार्बोफ्यूरेन,  फोरेट और ट्राइऐजोफॉस कृषि में इस्तेमाल होने वाले कीटनाशक हैं। केरल में जैविक खेती को बढ़ावा देने के लिये, राज्य के कृषि विभाग ने कीटनाशकों के उपयोग पर प्रतिबंध लगाने का आदेश दिया था। केरल कृषि विश्वविद्यालय को प्रतिबंधित कीटनाशकों जिसमें कार्बोफ्यूरेन, फोरेट, मिथाइल पैराथियाॅन, मोनोक्रोटोफॉस, मिथाइल डेमेथॉन आदि शामिल हैं के विकल्प प्रदान करने के लिये कहा गया था। विश्वविद्यालय ने कम खतरनाक कीटनाशकों का सुझाव दिया, जैसे एसीफेट, कार्बेरिल, डाइमेथोएट और फ्लुबेंडियामाइड। अतः विकल्प A सही है।

Source: https://www.downtoearth.org.in/news/pesticide-ban-lands-kerala-in-court-33657

Covered in Drishti: https://www.drishtiias.com/daily-updates/daily-news-analysis/pesticide-ban


40. निम्नलिखित कथनों पर विचार कीजिये:

1. रामसर सम्मेलन के अनुसार, भारत के राज्यक्षेत्र में सभी आर्द्र भूमियों को बचाना और संरक्षित रखना भारत सरकार के लिये अधिदेशात्मक है।
2. आर्र्द भूमि (संरक्षण और प्रबंधन) नियम, 2010, भारत सरकार ने रामसर सम्मेलन की संस्तुतियों के आधार पर बनाए थे।
3. आर्द्र भूमि (संरक्षण और प्रबंधन) नियम, 2010, आर्द्र भूमियों के अपवाह क्षेत्र या जलग्रहण क्षेत्रों को भी सम्मिलित करते हैं, जैसा कि प्राधिकार द्वारा निर्धारित किया गया है।

उपर्युक्त में से कौन-सा/से कथन सही है/हैं?

(a) केवल 1 और 2

(b) केवल 2 और 3

(c) केवल 3         

(d) 1, 2 और 3

उत्तर: (b/c)

व्याख्या: कथन 1 सही नहीं है- रामसर सम्मेलन के तहत भारत सरकार के लिये केवल रामसर स्थलों बचाना और संरक्षित रखना भारत सरकार के लिये अधिदेशात्मक है  न कि भारत के राज्यक्षेत्र में अवस्थित सभी आर्द्र भूमियों  को।

कथन 3 सही है- आर्द्र भूमि (संरक्षण और प्रबंधन ) नियम, 2010 के अंतर्गत, आर्द्र भूमि" का अर्थ उन  क्षेत्रों से है जहाँ जल पर्यावरण और संबंधित वनस्पति एवं जंतु जीवन को नियंत्रित करने वाला प्राथमिक कारक होता है। यह वहाँ ऐसे स्थानों पर पाई जातीं हैं जहाँ जल स्तर भूमि की सतह पर या उसके निकट होता है अथवा जहाँ भूमि जल से आप्लावित होती है। यह प्राकृतिक या कृत्रिम, स्थायी या अस्थायी, स्थिर या प्रवाहमान जल वाले तथा लवणीय या अलवणीय हो सकते हैं एवं समुद्री जल के क्षेत्रों सहित इनकी गहराई कम ज्वार पर छह मीटर से अधिक नहीं होती है और इसमें सभी अंतर्देशीय जलीय क्षेत्र जैसे झीलें, जलाशय, तालाब, पाश्चजल, लैगून, खाड़ी, नदी के मुहाने और मानव निर्मित आर्द्रभूमि आदि क्षेत्र शामिल होते हैं। आर्द्र भूमियों के अपवाह क्षेत्र या जलग्रहण क्षेत्रों को भी सम्मिलित किया गया है, जैसा कि प्राधिकार द्वारा निर्धारित किया गया है किंतु इसमें मुख्य नदी चैनल, धान के खेत और तटीय आर्द्र भूमि शामिल नहीं हैं।

कथन 2 की सत्यता पर भ्रम के कारण, हम अभी निश्चित निष्कर्ष पर नहीं पहुँचे हैं।

source:

https://www.drishtiias.com/hindi/quiz/quizlist/60-steps-to-prelims

https://www.drishtiias.com/hindi/summary-of-important-reports/cbd-sixth-national-report

https://www.drishtiias.com/hindi/daily-updates/daily-news-analysis/world-wetland-day

https://www.drishtiias.com/hindi/daily-updates/daily-news-editorials/importance-of-wetlands


41. निम्नलिखित कथनों पर विचार कीजिये:

1. कृषि मृदाएँ पर्यावरण में नाइट्रोजन के ऑक्साइड निर्मुक्त करती हैं।
2. मवेशी पर्यावरण में अमोनिया निर्मुक्त करते हैं।
3. कुक्कुट उद्योग पर्यावरण में अभिक्रियाशील नाइट्रोजन यौगिक निर्मुक्त करते हैं।

उपर्युक्त में से कौन-सा/से सही है/हैं?

(a) केवल 1 और 3

(b) केवल 2 और 3

(c) केवल 2         

(d) 1, 2 और 3

उत्तर: (d)

व्याख्या: कथन 1 सही है: वर्ष 2010 में भारत से नाइट्रोजन के ऑक्साइड उत्सर्जन में कृषि मृदाओं का योगदान 70% से अधिक था, जिसके बाद अपशिष्ट जल (12%) और आवासीय तथा वाणिज्यिक गतिविधियों (6%) का स्थान था।

कथन 2 सही है: अमोनिया उत्पादन में मवेशियों की भागीदारी 80% है और भारत विश्व स्तर पर अमोनिया उत्सर्जन का सबसे बड़ा स्रोत है, जो नाइट्रोजन के ऑक्साइड के उत्सर्जन से लगभग दोगुना है।

कथन 3 सही है: कुक्कुट उद्योग ने 6% की वार्षिक वृद्धि दर के साथ वर्ष 2016 में 0.415 टन अभिक्रियाशील नाइट्रोजन यौगिकों का उत्सर्जन किया।

source:

https://www.drishtiias.com/hindi/daily-updates/daily-news-analysis/nitrogen-emissions-going-up-study


42. अलियार, इसापुर और कंग्साबती जैसे ज्ञात स्थानों में क्या समानता  है?

(a) हाल ही में खोजे गए यूरेनियम निक्षेप

(b) उष्णकटिबंधीय वर्षावन

(c) भूमिगत गुफा तंत्र

(d) जल भंडार

उत्तर: (d)

व्याख्या: अलियार (तमिलनाडु) इसापुर (महाराष्ट्र) और कंग्साबती (पश्चिम बंगाल) ऐसे जल भंडार हैं, जहाँ जल स्तर सामान्य स्तर से काफी नीचे पहुँच गया है।

Source: https://www.downtoearth.org.in/news/water/water-level-in-91-reservoirs-at-36-per-cent-of-total-capacity-southern-states-most-affected-59826


43. सार्वजनिक परिवहन में बसों के लिये ईंधन के रूप में हाइड्रोजन संवर्द्धित CNG (H-CNG) का इस्तेमाल करने के प्रस्तावों के संदर्भ में, निम्नलिखित कथनों पर विचार कीजिये:

1. H-CNG के इस्तेमाल का मुख्य लाभ कार्बन मोनोक्साइड के उत्सर्जनों का विलोपन है।
2. ईंधन के रूप में H-CNG कार्बन डाइऑक्साइड और हाइड्रोकार्बन उत्सर्जनों को कम करती है।
3. बसों के लिये ईंधन के रूप में CNG के साथ हाइड्रोजन को आयतन के आधार पर पाँचवें हिस्से तक मिलाया जा सकता  है।
4. CNG की अपेक्षा H-CNG ईंधन को कम खर्चीला बनाती  है।

उपर्युक्त में से कौन-सा/से कथन सही है/हैं?

(a) केवल 1

(b) केवल 2 और 3

(c) केवल 4

(d) 1, 2, 3 और 4

उत्तर: (b)

व्याख्या: हाइड्रोजन संवर्द्धित CNG (H-CNG) हाइड्रोजन और CNG का मिश्रण है, जिसमे आदर्श हाइड्रोजन सांद्रता 18% है। अतः कथन 3 सही है।

परंपरागत CNG की तुलना में, H-CNG का उपयोग कार्बन मोनोऑक्साइड के उत्सर्जन को 70% तक कम कर सकता है, किंतु इससे इसका विलोपन संभव नहीं हैं। अतः कथन 1 सही नहीं है। यह कार्बन डाइऑक्साइड और हाइड्रोकार्बन उत्सर्जन को भी कम करता है। अतः कथन 2 सही है।

साथ ही H-CNG का उपयोग ईंधन दक्षता में 5% तक की बचत को सक्षम बनाता है। H-CNG इंजन के बिना जले हाइड्रोकार्बन उत्सर्जन को भी कम करता है और इंजन की दहन प्रक्रिया (Combustion) को तेज़ करता है।

Source: https://indianexpress.com/article/explained/cng-to-hydrogen-cng-why-switch-and-how-5278356/


44. मेघाच्छादित रात में ओस की बूँदें क्यों नहीं बनतीं?

(a) भूपृष्ठ से निर्मुक्त विकिरण को बादल अवशोषित कर लेते हैं।

(b) पृथ्वी के विकिरण को बादल वापस परावर्तित कर देते हैं।

(c) मेघाच्छादित रातों में भूपृष्ठ का तापमान कम होता है।

(d) बादल बहते हुए पवन को भूमितल की ओर विक्षेपित कर देते हैं।

उत्तर: (b)

व्याख्या: ओस वातावरण में फैले हुए वाष्प का वह रूप है जो जमकर जलबिंदु अथवा छोटी-छोटी बूँदों के रूप में परिवर्तित होकर पृथ्वी पर गिरता है। ओस बनने की प्रक्रिया का संघनन से सीधा जुड़ाव है। वातावरण में शत-प्रतिशत सापेक्षिक आर्द्रता होने पर वायु संतृप्त हो जाती है और संघनन आरंभ हो जाता है। संतृप्तावस्था के बाद वायु को ठंडा करने पर संघनन प्रारंभ हो जाता है। शत-प्रतिशत सापेक्षिक आर्द्रता के बाद अतिरिक्त आर्द्रता का संघनन हो जाता है। ओस के अनुकूल मौसमी तत्त्वों में साफ आसमान, हल्की हवा, मृदा की नमी शामिल हैं।

ओस तब बनती है जब तापमान ओसांक के बराबर हो जाता है जिस तापमान पर हवा संतृप्त होती है, उसे ओसांक या ओस बिंदु कहते हैं। यह अक्सर दो कारणों से पहले भूमि स्तर पर होता है। सबसे पहले, दीर्घ तरंग उत्सर्जन के कारण रात में पृथ्वी की सतह शीतल हो जाती है। बादल इसे होने से रोकते हैं क्योंकि यह पृथ्वी के विकिरण को परावर्तित करता है।

संघनन के लिये तापमान को ओस बिंदु तक कम करने की आवश्यकता होती है। दूसरा, मृदा अक्सर ओस के लिये नमी का स्रोत होती है। ऊष्म और नम मृदा ओस के निर्माण में मदद करेगी क्योंकि मृदा रात भर शीतल रहती है।

Source: https://www.weather.gov/source/zhu/ZHU_Training_Page/fog_stuff/Dew_Frost/Dew_Frost.htm


45. निम्नलिखित कथनों पर विचार कीजिये:

1. भारत के संविधान के 44वें संशोधन द्वारा लाए गए एक अनुच्छेद ने प्रधानमंत्री के निर्वाचन को न्यायिक पुनर्विलोकन के परे कर दिया।
2. भारत के संविधान के 99वें संशोधन को भारत के उच्चतम न्यायालय ने अभिखंडित कर दिया क्योंकि यह न्यायपालिका की स्वतंत्रता का अतिक्रमण करता था।

उपर्युक्त में से कौन-सा/से कथन सही है/हैं?

(a) केवल 1       

(b) केवल 2

(c) 1 और 2 दोनों

(d) न तो 1, न ही 2

उत्तर: (b)

व्याख्या: वर्ष 1975 में, संसद द्वारा संविधान में उनतालीसवाँ संशोधन किया गया था, जिसने राष्ट्रपति, उपराष्ट्रपति, प्रधानमंत्री और लोकसभा अध्यक्ष के निर्वाचन के संबंध में याचिकाओं पर निर्णय लेने के लिये उच्चतम न्यायालय के अधिकार को समाप्त कर दिया था और इसके स्थान पर एक ऐसे निकाय में चुनावी विवादों को हल करने की शक्ति निहित की थी जिसका गठन संसद द्वारा किया जाना था।

अत: विकल्प 1 गलत है।

 भारत के संविधान के 99वें संशोधन को भारत के उच्चतम न्यायालय ने अभिखंडित कर दिया क्योंकि यह "न्यायपालिका की स्वतंत्रता" के सिद्धांतों के साथ-साथ "शक्तियों के पृथक्करण" के सिद्धांत का उल्लंघन भी करता है

source:

https://www.drishtiias.com/hindi/loksabha-rajyasabha-discussions/institutional-crisis-in-judiciary

https://www.drishtiias.com/hindi/loksabha-rajyasabha-discussions/conspiracy-against-judiciary

https://www.drishtiias.com/hindi/daily-updates/daily-news-editorials/major-concerns-related-to-the-country-judiciary


46. निम्नलिखित कथनों पर विचार कीजिये:

1. न्यायाधीश (जाँच) अधिनियम, 1968 के अनुसार, भारत के उच्चतम न्यायालय के किसी न्यायाधीश पर महाभियोग चलाने के प्रस्ताव को लोकसभा के अध्यक्ष द्वारा अस्वीकार नहीं किया जा सकता।
2. भारत का संविधान यह परिभाषित करता है और ब्यौरे देता है कि क्या-क्या भारत के उच्चतम न्यायालय के न्यायाधीशों की ‘अक्षमता और सिद्ध कदाचार’ को गठित करते हैं।
3. भारत के उच्चतम न्यायालय के न्यायाधीशों के महाभियोग की प्रक्रिया के ब्यौरे न्यायाधीश (जाँच) अधिनियम, 1968 में दिये गए हैं।
4. यदि किसी न्यायाधीश के महाभियोग के प्रस्ताव को मतदान हेतु लिया जाता है, तो विधि द्वारा अपेक्षित है कि यह प्रस्ताव संसद के प्रत्येक सदन द्वारा समर्थित हो और उस सदन की कुल सदस्य संख्या के बहुमत द्वारा तथा संसद के उस सदन के कुल उपस्थित और मत देने वाले सदस्यों के कम-से-कम दो-तिहाई द्वारा समर्थित हो।

उपर्युक्त में से कौन-सा/से कथन सही है/हैं?

(a) 1 और 2       

(b) केवल 3

(c) केवल 3 और 4

(d) 1, 3 और 4

उत्तर: (c)

व्याख्या: . न्यायाधीश (जाँच) अधिनियम, 1968 महाभियोग की प्रक्रिया द्वारा उच्चतम न्यायालय के एक न्यायाधीश को हटाने से संबंधित प्रक्रिया का ब्यौरा डेता है। अतः कथन 3 सही है।

  1. महाभियोग प्रस्ताव पेश करने के लिये लोकसभा में 100 सांसदों और राज्यसभा में 50 सदस्यों के हस्ताक्षर युक्त महाभियोग प्रस्ताव की ज़रूरत होती है।
  2. राज्यसभा चेयरमैन या लोकसभा अध्यक्ष पर यह निर्भर करता है कि वह इस प्रस्ताव पर क्या निर्णय लेते हैं। इसे मंज़ूर भी किया जा सकता है और नामंज़ूर भी। अतः कथन 1 सही नहीं है।
  3. यदि राज्यसभा चेयरमैन या लोकसभा अध्यक्ष इस प्रस्ताव को मंज़ूर कर लेते हैं तो आरोपों की जाँच के लिये तीन सदस्यीय समिति का गठन किया जाता है।
  4. समिति में (a) मुख्य न्यायाधीश या उच्चतम न्यायालय के न्यायाधीश, (b) उच्च न्यायालय के मुख्य न्यायाधीश, और (c) एक प्रतिष्ठित न्यायविद शामिल होना चाहिये
  5. यदि समिति न्यायाधीश को दुर्व्यवहार का दोषी या असक्षम पाती है, तो सदन प्रस्ताव पर विचार कर सकता है।
  6. संसद के प्रत्येक सदन द्वारा विशेष बहुमत से प्रस्ताव पारित होने के बाद, न्यायाधीश को हटाने के लिये राष्ट्रपति के पास भेजा जाता है। अतः कथन 4 सही है। कथन 1 गलत है और 4 सही है विलोपन से हम देख सकते हैं कि केवल विकल्प C सही है।
  7. अंत में, राष्ट्रपति न्यायाधीश को हटाने का आदेश जारी कर देते है।

कथन 2 सही नहीं है: संविधान कहीं भी "अक्षमता और सिद्ध दुर्व्यवहार" शब्द को परिभाषित नहीं करता है।

source:

https://www.drishtiias.com/hindi/loksabha-rajyasabha-discussions/impeachement

https://www.drishtiias.com/hindi/loksabha-rajyasabha-discussions/impeachement-of-cji


47. किस प्रधानमंत्री के कार्यकाल के दौरान भारत के संविधान में नौवीं अनुसूची को पुर:स्थापित किया गया था?

(a) जवाहरलाल नेहरू

(b) लाल बहादुर शास्त्री

(c) इंदिरा गांधी

(d) मोरारजी देसाई

उत्तर: (a)

व्याख्या: कामेश्वर सिंह बनाम बिहार राज्य के निर्णय के उपरांत, सरकार को आशंका हुई कि पूरे कृषि सुधार कार्यक्रम खतरे में पड़ सकते हैं और ऐसा न हो यह सुनिश्चित करने के लिये संसद ने वर्ष 1951 में जवाहर लाल नेहरू के प्रधानमंत्रित्त्व काल में संविधान में संशोधन किया, जिसने संविधान में नौवीं अनुसूची को समाविष्ट किया।

source:

https://www.drishtiias.com/hindi/loksabha-rajyasabha-discussions/9th-schedule-and-sc-st-act


48. निम्नलिखित कथनों पर विचार कीजिये:

1. भारत सरकार द्वारा कोयला क्षेत्र का राष्ट्रीयकरण इंदिरा गांधी के कार्यकाल में किया गया था।
2. वर्तमान में, कोयला खंडों का आवंटन लॉटरी के आधार पर किया जाता है।
3. भारत हाल के समय तक घरेलू आपूर्ति की कमी को पूरा करने के लिये कोयले का आयात करता था, किंतु अब भारत कोयला उत्पादन में आत्मनिर्भर है।

उपर्युक्त में से कौन-सा/से कथन सही है/हैं?

(a) केवल 1

(b) केवल 2 और 3

(c) केवल 3

(d) 1, 2 और 3

उत्तर: (a)

व्याख्या: 

  1. वर्ष 1972 में इंदिरा गांधी प्रधानमंत्रित्त्व काल में कोयला क्षेत्र का दो चरणों में राष्ट्रीयकरण किया गया था।
  2. कोयला खंडो का आवंटन नीलामी के माध्यम से किया जाता है न कि लॉटरी के आधार पर।
  3. कोयला क्षेत्र भारत में एकाधिकार क्षेत्र है। भारत के पास विश्व का 5वाँ सबसे बड़ा कोयला भंडार है, लेकिन एकाधिकार फर्मों द्वारा कोयला उत्पादन की अक्षमता के कारण, यह घरेलू आपूर्त्ति की कमी को पूरा करने के लिये कोयले का आयात करता है। लेकिन, अभी भी देश कोयला उत्पादन में आत्मनिर्भर नहीं है।

अतः विकल्प (A) सही है।

source:

https://www.drishtiias.com/hindi/daily-updates/daily-news-analysis/an-index-to-determine-the-value-of-coal-blocks


49. निम्नलिखित कथनों पर विचार कीजिये:

1. संसद (निरर्हता निवारण) अधिनियम, 1959 कई पदों को ‘लाभ का पद’ के आधार पर निरर्हता से छूट देता है।
2. उपर्युक्त अधिनियम पाँच बार संशोधित किया गया था।
3. शब्द ‘लाभ का पद’ भारत के संविधान में भली-भाँति परिभाषित किया गया है।

उपर्युक्त में से कौन-सा/से कथन सही है/हैं?

(a) केवल 1 और 2

(b) केवल 3

(c) केवल 2 और 3

(d) 1, 2 और 3

उत्तर: (a)

व्याख्या: कथन 1:- संसद (निरर्हता निवारण) अधिनियम, 1959 कई पदों को ‘लाभ का पद’ के आधार पर निरर्हता से छूट देता है।

जैसे कि:-

(i) राज्यमंत्री और उपमंत्री,

(ii) संसदीय सचिव और संसदीय अवर सचिव,

(iii) संसद में उप मुख्य सचेतक,

(iv) विश्वविद्यालयों के कुलपति,

(v) राष्ट्रीय कैडेट कोर और प्रादेशिक सेना के अधिकारी, और

(vi) सरकार द्वारा गठित सलाहकार समितियों के अध्यक्ष और सदस्य यदि वे क्षतिपूर्ति आदि के अतिरिक्त किसी शुल्क या पारिश्रमिक के हकदार नहीं हैं। अतः कथन 1 सही है।

कथन 2:- इस अधिनियम को वर्ष 1960, 1992,1993,2006 और 2013 में अब तक 5 बार संशोधित किया जा चुका है। अतः कथन 2 सही है।

कथन 3:- लाभ के पद को भारतीय कानून या संविधान स्पष्ट रूप से परिभाषित नहीं करता है किंतु यह परिभाषा वर्षों में विभिन्न न्यायिक निर्णयों में की गई व्याख्याओं के साथ विकसित हुई है। अतः कथन 3 गलत है। इस सूचना के आधार पर हम विकल्प b, c और d को विलोपित कर सकते हैं।

source:

https://www.drishtiias.com/hindi/mains-practice-question/question-685

https://www.drishtiias.com/hindi/daily-updates/daily-news-editorials/explained-law-on-holding-an-office-of-profit

https://www.drishtiias.com/hindi/daily-updates/daily-news-analysis/disqualification-of-aap-mlas-approved


50. भारत के संविधान की किस अनुसूची के अधीन जनजातीय भूमि का, खनन के लिये निजी पक्षकारों को अंतरण अकृत और शून्य घोषित किया जा सकता है?

(a) तीसरी अनुसूची

(b) पाँचवीं अनुसूची

(c) नौवीं अनुसूची   

(d) बारहवीं अनुसूची

उत्तर: (b)

व्याख्या: अनुसूचित क्षेत्रों में खनिज रियायतों की स्वीकृति की घोषणा, अनुसूचित क्षेत्रों और जनजातीय क्षेत्रों के प्रशासन से संबंधित संविधान की पाँचवीं एवं छठी अनुसूचियों के साथ-साथ अनुच्छेद 244 में निहित प्रावधानों तथा पंचायतों का विस्तार(अनुसूचित क्षेत्रों तक विस्तार) अधिनियम,1996 और अनुसूचित जनजाति और अन्य परंपरागत वन निवासी (वन अधिकारों की मान्यता) अधिनियम, 2006 तथा आदिवासियों के हितों की रक्षा करने वाले अन्य प्रासंगिक वैधानिक अधिनियमों के प्रावधानों द्वारा निर्देशित है।

पाँचवीं अनुसूची के अंतर्गत, राज्यपाल सार्वजनिक अधिसूचना द्वारा निर्देश दे सकते हैं कि संसद या राज्य के विधानमंडल का कोई विशेष अधिनियम राज्य में अनुसूचित क्षेत्र या उसके किसी हिस्से पर लागू होगा या नहीं होगा।

source:

https://www.drishtiias.com/hindi/mains-practice-question/question-1491


51. भारत में विशिष्टत: असुरक्षित जनजातीय समूहों ख्पर्टिकुलरली वल्नरेबल ट्राइबल ग्रुप्स (PVTGs)] के बारे में, निम्नलिखित कथनों पर विचार कीजिये:

1. PVTGs देश के 18 राज्यों तथा एक संघ राज्यक्षेत्र में निवास करते हैं।
2. स्थिर या कम होती जनसंख्या, PVTG स्थिति के निर्धारण के मानदंडों में से एक है।
3. देश में अब तक 95 PVTGs आधिकारिक रूप से अधिसूचित  हैं।
4. PVTGs की सूची में ईरूलार और कोंडा रेड्डी जनजातियाँ शामिल की गई हैं।

उपर्युक्त में से कौन-से कथन सही हैं?

(a) 1, 2 और 3

(b) 2, 3 और 4

(c) 1, 2 और 4

(d) 1, 3 और 4

उत्तर: (c)

व्याख्या: वर्तमान में देश में 95 नहीं बल्कि 75 असुरक्षित जनजातीय समूह (PVTG) अधिसूचित हैं। PVTG के निर्धारण के लिये अपनाए जाने वाले मानदंड निम्नानुसार हैं:

  1. प्रौद्योगिकी के पूर्व-कृषि स्तर  
  2. एक स्थिर या कम होती जनसंख्या;
  3. साक्षरता का निम्न स्तर और
  4. आर्थिक रूप से पिछड़ापन । अतः कथन 3 गलत है, जो हमें विलोपन विधि द्वारा सही उत्तर के रूप में विकल्प C देता है।

source

करेंट अफेयर्स टुडे, फरवरी- 2019 , पेज न. 33


52. भारत के संविधान के सदंर्भ में, सामान्य विधियों में अंतर्विष्ट प्रतिषेध अथवा निर्बंधन अथवा उपबंध, अनुच्छेद 142 के अधीन सांविधानिक शक्तियों पर प्रतिषेध अथवा निर्बंधन की तरह कार्य नहीं कर सकते। निम्नलिखित में से कौन-सा एक, इसका अर्थ हो सकता है?

(a) भारत के निर्वाचन आयोग द्वारा अपने कर्तव्यों का निर्वहन करते समय लिये गए निर्णयों को किसी भी न्यायालय में चुनौती नहीं दी जा सकती।

(b) भारत का उच्चतम न्यायालय अपनी शक्तियों के प्रयोग में संसद द्वारा निर्मित विधियों से बाध्य नहीं होता।

(c) देश में गंभीर वित्तीय संकट की स्थिति में, भारत का राष्ट्रपति मंत्रिमंडल के परामर्श के बिना वित्तीय आपात घोषित कर सकता है।

(d) कुछ मामलों में राज्य विधानमंडल, संघ विधानमंडल की सहमति के बिना, विधि निर्मित नहीं कर सकते।

उत्तर: (b)

व्याख्याः अनुच्छेद 142- उच्चतम न्यायालय की डिक्रियों और आदेशों का प्रवर्तन एवं प्रकटीकरण आदि के बारे में आदेश (1) उच्चतम न्यायालय अपनी अधिकारिता का प्रयोग करते हुए ऐसी डिक्री पारित कर सकेगा या ऐसा आदेश कर सकेगा जो उसके समक्ष लंबित किसी वाद या विषय में पूर्ण न्याय करने के लिये आवश्यक हो और इस प्रकार पारित डिक्री या किया गया आदेश भारत के राज्यक्षेत्र में सर्वत्र ऐसी रीति से, जो संसद‌ द्वारा बनाई गई किसी विधि द्वारा या उसके अधीन विहित की जाए और जब तक इस निमित्त इस प्रकार उपबंध नहीं किया जाता है तब तक, ऐसी रीति से जो राष्ट्रपति आदेश द्वारा विहित करे, प्रवर्तनीय होगा

source:

https://www.drishtiias.com/hindi/daily-updates/daily-news-analysis/govt-stand-on-lokpal-appointment-is-wholly-unsatisfactory-says-supreme-court

https://www.drishtiias.com/hindi/daily-updates/daily-news-editorials/article-142-and-the-need-for-judicial-restraint

https://www.drishtiias.com/hindi/mains-practice-question/question-521


53. भारत के किसी राज्य की विधान सभा के संदर्भ में, निम्नलिखित कथनों पर विचार कीजिये:

1. वर्ष के प्रथम सत्र के प्रारंभ में राज्यपाल सदन के सदस्यों के लिये रूढ़िगत संबोधन करता है।
2. जब किसी विशष्ट विषय पर राज्य विधानमंडल के पास कोई नियम नहीं होता, तो उस विषय पर वह लोकसभा के नियम का पालन करता है।

उपर्युक्त में से कौन-सा/से कथन सही है/हैं?

(a) केवल 1       

(b) केवल 2

(c) 1 और 2 दोनों

(d) न तो 1, न ही 2

उत्तर: (a)

व्याख्या: कथन 1 सही है: अनुच्छेद 176(1) राज्यपाल, विधानसभा के लिये प्रत्येक साधारण निर्वाचन के पश्चात् प्रथम सत्र के आरम्भ में और प्रत्येक वर्ष के प्रथम सत्र के आरम्भ में विधानसभा में या विधान परिषद वाले राज्य की दशा में एक साथ समवेत दोनों सदनों में अभिभाषण करेगा और विधान-मण्डल को उसके आवाहन के कारण बताएगा।

कथन 2 सही नही है: विशेष विषयों में नियमों की अनुपस्थिति के मामले में राज्यपाल, विधान परिषद वाले राज्य में विधानसभा के अध्यक्ष और विधान परिषद के सभापति से परामर्श करने के पश्चात् दोनों सदनों में परस्पर संचार से सम्बन्धित प्रक्रिया के नियम बना सकेगा

अनुच्छेद 208 – प्रक्रिया के नियम

अनुच्छेद 208(1) किसी राज्य के विधानमंडल का कोई सदन इस संविधान के प्रावधानों, इसकी प्रक्रिया और अपने कार्य के संचालन के अधीन विनियमन के लिये नियम बना सकता है।

अनुच्छेद 208(2) जब तक खंड (1) के अधीन नियम नहीं बनाए जाते हैं तब तक इस संविधान के प्रारंभ से ठीक पहले तत्स्थानी प्रांत के विधान-मंडल के संबंध में जो प्रक्रिया के नियम और स्थायी आदेश प्रवृत्त थे वे ऐसे उपांतरणों और अनुकूलनों के अधीन रहते हुए उस राज्य के विधान-मंडल के संबंध में प्रभावी होंगे जिन्हें, यथास्थिति, विधानसभा का अध्यक्ष या विधान परिषद् का सभापति उनमें करे।

अनुच्छेद 208(3) राज्यपाल, विधान परिषद वाले राज्य में विधानसभा के अध्यक्ष और विधान परिषद के सभापति से परामर्श करने के पश्चात् दोनों सदनों में परस्पर संचार से सम्बन्धित प्रक्रिया के नियम बना सकेगा।

source: 

 https://indiankanoon.org/doc/695063/

https://www.drishtiias.com/hindi/daily-updates/daily-news-editorials/privileges-of-legislative

https://www.drishtiias.com/hindi/loksabha-rajyasabha-discussions/karnataka-discretionary-powers-of-the-governor


54. निम्नलिखित कथनों पर विचार कीजिये:

1. भ्रष्टाचार के विरुद्ध संयुक्त राष्ट्र कन्वेंशन ख्यूनाइटेड नेश्ंास कन्वेंशन अगेंस्ट करप्शन (UNCAC)] का ‘भूमि, समुद्र और वायुमार्ग से प्रवासियों की तस्करी के विरुद्ध एक प्रोटोकॉल’ होता है।
2. UNCAC अब तक का सबसे पहला विधित: बाध्यकारी सार्वभौम भ्रष्टाचार-निरोधी लिखत है।
3. राष्ट्र-पार संगठित अपराध के विरुद्ध संयुक्त राष्ट्र कन्वेंशन (यूनाइटेड नेशंस कन्वेंशन अगेंस्ट ट्रांसनैशनल ऑर्गेनाइज़्ड क्राइम (UNTOC)] की एक विशिष्टता ऐसे एक विशिष्ट अध्याय का समावेशन है, जिसका लक्ष्य उन संपत्तियों को उनके वैध स्वामियों को लौटाना है जिनसे वे अवैध तरीके से ले ली गई थीं।
4. मादक द्रव्य और अपराध विषयक संयुक्त राष्ट्र कार्यालय ख्यूनाइटेड नेशंस ऑफिस ऑन ड्रम्स एंड क्राइम (UNODC)] संयुक्त राष्ट्र के सदस्य राज्यों द्वारा UNCAC और UNTOC दोनों के कार्यान्वयन में सहयोग करने के लिये अधिदेशित है।

उपर्युक्त में से कौन-सा/से कथन सही है/हैं?

(a) केवल 1 और 3

(b) केवल 2, 3 और 4

(c) केवल 2 और 4

(d) 1, 2, 3 और 4

उत्तर: (c)

व्याख्या: भूमि, समुद्र और वायु मार्ग से प्रवासियों की तस्करी के विरुद्ध प्रोटोकॉल, यह प्रोटोकॉल संयुक्त राष्ट्र संघ के अंतर्राष्ट्रीय संगठित अपराध के विरुद्ध अभिसमय (UNTOC) के अंतर्गत आता है। इसका मुख्य उद्देश्य अंतर्राष्ट्रीय स्तर पर संगठित अपराध के विरुद्ध कार्यवाई करना है। अतः कथन 1 गलत है।

(ii) भ्रष्टाचार के विरुद्ध संयुक्त राष्ट्र अभिसमय (UNCAC) एकमात्र कानूनी रूप से बाध्यकारी सार्वभौमिक भ्रष्टाचार-विरोधी कानून है। अभिसमय में भ्रष्टाचार के कई रूपों, जैसे रिश्वतखोरी, प्रभाव में व्यापार (इंफ्लुएंस पेड्लिंग), शक्तियों का दुरुपयोग और निजी क्षेत्र में भ्रष्टाचार के विभिन्न कार्य आदि को शामिल किया गया है। अतः कथन 2 सही है।

(iii) "संपत्तियों को उनके वास्तविक स्वामियों को लौटाना जिनसे वे अवैध रूप से ली गई हैं", भ्रष्टाचार के विरुद्ध संयुक्त राष्ट्र अभिसमय (UNCAC) के अंतर्गत शामिल है। अतः कथन 3 गलत है कथन 3 वाले विकल्पों को हटाने से भी हमें सही उत्तर मिल जाता है, वह विकल्प C सही है।

(iv)  मादक द्रव्य और अपराध विषयक संयुक्त राष्ट्र कार्यालय यूनाइटेड नेशंस ऑफिस ऑन ड्रम्स एंड क्राइम (UNODC)] संयुक्त राष्ट्र के सदस्य राज्यों द्वारा UNCAC और UNTOC दोनों के कार्यान्वयन में सहयोग करने के लिये अधिदेशित है, साथ ही वर्ष 1961, 1971 और 1988 के संयुक्त राष्ट्र मादक द्रव्य तथा अपराध विषयक अभिसमयों के साथ-साथ UNODC के सभी परिचालन कार्यों को रेखांकित करता है। अतः कथन 4 सही है।

अतः विकल्प (C) सही है।

source:

https://www.drishtiias.com/hindi/daily-updates/daily-news-analysis/home-the-most-dangerous-place-for-women


55. निम्नलिखित कथनों पर विचार कीजिये:

1. भारतीय वन अधिनियम, 1927 में हाल में हुए संशोधन के अनुसार, वन निवासियों को वनक्षेत्रों में उगने वाले बाँस को काट गिराने का अधिकार है।
2. अनुसूचित जनजाति एवं अन्य पारंपरिक वनवासी (वन अधिकारों की मान्यता) अधिनियम, 2006 के अनुसार, बाँस एक गौण वनोपज है।
3. अनुसूचित जनजाति एवं अन्य पारंपरिक वनवासी (वन अधिकारों की मान्यता) अधिनियम, 2006, वन निवासियों को गौण वनोपज के स्वामित्व की अनुमति देता है।

उपर्युक्त में से कौन-सा/से कथन सही है/हैं?

(a) केवल 1 और 2

(b) केवल 2 और 3

(c) केवल 3         

(d) 1, 2 और 3

उत्तर: (b)

व्याख्या: भारतीय वन (संशोधन) विधेयक, 2017 गैर-वन क्षेत्रों में उगाए जाने वाले बाँस की कटाई और पारगमन की अनुमति देता है। हालाँकि वन भूमि पर उगाए गए बाँस को एक वृक्ष के रूप में वर्गीकृत किया जाना जारी रहेगा और मौजूदा कानूनी प्रतिबंधों द्वारा निर्देशित किया जाएगा। अत: कथन 1 सही नहीं है।

अनुसूचित जनजाति और अन्य पारंपरिक वन निवासी (वन अधिकारों की मान्यता) अधिनियम, 2006 बाँस को लघु वन उपज के रूप में मान्यता देता है तथा अनुसूचित जनजातियों एवं पारंपरिक वन निवासियों को "स्वामित्त्व, लघु वन उपज एकत्र करने, उपयोग और निपटान तक पहुँच" का अधिकार देता है। अत: कथन 2 और 3 सही हैं।

source:

https://www.drishtiias.com/hindi/daily-updates/daily-news-analysis/the-bamboo-curtain

https://www.drishtiias.com/hindi/mains-practice-question/question-1746


56. भारत के संविधान का कौन-सा अनुच्छेद अपनी पसंद के व्यक्ति से विवाह करने के किसी व्यक्ति के अधिकार को संरक्षण देता है?

(a) अनुच्छेद 19

(b) अनुच्छेद 21

(c) अनुच्छेद 25

(d) अनुच्छेद 29

उत्तर: (b)

source:

व्याख्या: एक व्यक्ति द्वारा अपनी पसंद के किसी व्यक्ति से विवाह करने का अधिकार संविधान के अनुच्छेद-21 का अभिन्न अंग है। जो कहता है, "किसी भी व्यक्ति को विधि द्वारा स्थापित प्रक्रिया के अतिरिक्त उसके जीवन और वैयक्तिक स्वतंत्रता के अधिकार से वंचित नहीं किया जा सकता है"। विवाह के अधिकार के संदर्भ में कुछ भारतीय मामलों का उल्लेख किया जा सकता है। विवाह से पूर्व यौन रोग से पीड़ित व्यक्ति को तब तक विवाह करने का कोई अधिकार नहीं दिया जा सकता है जब तक कि वह रोग से पूर्णतः ठीक नहीं हो जाता।

लता सिंह बनाम उत्तर प्रदेश राज्य 2006 वाद में, उच्चतम न्यायालय ने भारतीय संविधान के अनुच्छेद 21 के अंतर्गत विवाह के अधिकार को जीवन के अधिकार का एक अभिन्न अंग माना, न्यायालय ने कहा कि:

"यह एक स्वतंत्र और लोकतांत्रिक देश है तथा एक बार एक व्यक्ति बालिग हो जाता है या वह जिसे भी पसंद करता है उससे विवाह कर सकता है। यदि लड़का या लड़की के माता-पिता इस तरह के अंतर-जातीय या अंतर-धार्मिक विवाह को मंजूरी नहीं देते हैं, तो वे अधिकतम यह कर सकते हैं कि वे बेटे या बेटी के साथ सामाजिक संबंधों को तोड़ सकते हैं, लेकिन वे उन्हें धमकी नहीं दे सकते हैं। कोई उस व्यक्ति को परेशान नहीं कर सकता है जो इस तरह के अंतर-जातीय या अंतर-धार्मिक विवाह से गुजरता है”।

source:

https://www.drishtiias.com/hindi/daily-updates/daily-news-analysis/choosing-a-partener-is-a-person-fundamental-right

https://www.drishtiias.com/hindi/to-the-points/paper4/ethical-side-of-homosexuality


57. निम्नलिखित कथनों पर विचार कीजिये:

1. भारतीय पेटेंट अधिनियम के अनुसार, किसी बीज को बनाने की जैव प्रक्रिया को भारत में पेटेंट कराया जा सकता है।
2. भारत में कोई बौद्धिक संपदा अपील बोर्ड नहीं है।
3. पादप किस्में भारत में पेटेंट कराए जाने के पात्र नहीं हैं।

उपर्युक्त में से कौन-सा/से कथन सही है/हैं?

(a) केवल 1 और 3

(b) केवल 2 और 3

(c) केवल 3         

(d) 1, 2 और 3

उत्तर: (c)

व्याख्या: भारतीय पेटेंट अधिनियम के अनुच्छेद 3(जे) में पेटेंट योग्यता से "पौधों और पशुओं को संपूर्ण या किसी भी हिस्से में सूक्ष्मजीवों के अलावा, बीज, किस्मों और प्रजातियों सहित पौधों और जीवों के उत्पादन या प्रवर्धन के लिये अनिवार्य रूप से जैव प्रक्रियाओं को शामिल नहीं किया गया है"। अतः कथन 1 सही नहीं है।

भारतीय ट्रेडमार्क अधिनियम, 1999 और माल का भौगोलिक उपदर्शन (रजिस्ट्रीकरण और संरक्षण) अधिनयम, 1999 के अंतर्गत रजिस्ट्रार के निर्णयों के विरुद्ध अपील सुनने और समाधान करने के लिये भारत सरकार द्वारा वर्ष 2003 में बौद्धिक संपदा अपील बोर्ड (IPAB) का गठन किया गया था। अतः कथन 2 गलत है

पादप विविधता संरक्षण पादप प्रजनक अधिकारों (PBR) के रूप में एक प्रजनक को पादप विविधता का कानूनी संरक्षण प्रदान करता है। इस संबंध में भारत में, पौधा किस्म और कृषक अधिकार संरक्षण अधिनियम, 2001 कानूनी संरक्षण प्रदान करता है।

इसका उद्देश्य पौधों की किस्मों की सुरक्षा और पौधों के प्रजनकों एवं कृषक अधिकार संरक्षण के लिये एक प्रभावी प्रणाली की स्थापना करना है। अतः कथन 3 सही है।

अतः विकल्प C सही है।

source:

https://www.drishtiias.com/hindi/daily-updates/daily-news-editorials/intellectual-property-rights-of-india-challenges-and-way-out

https://www.drishtiias.com/hindi/quiz/quizlist/60-steps-to-prelims


58. निम्नलिखित कथनों पर विचार कीजिये:

पर्यावरण संरक्षण अधिनियम, 1986 भारत को सशक्त करता है कि

1. वह पर्यावरणीय संरक्षण की प्रक्रिया में लोक सहभागिता की आवश्यकता का और इसे हासिल करने की प्रक्रिया और रीति का विवरण दे
2. वह विभिन्न स्रोतों से पर्यावरणीय प्रदूषकों के उत्सर्जन या विसर्जन के मानक निर्धारित करे

उपर्युक्त में से कौन-सा/से कथन सही है/हैं?

(a) केवल 1       

(b) केवल 2

(c) 1 और 2 दोनों

(d) न तो 1, न ही 2

उत्तर: (c)

व्याख्या:  पर्यावरण संरक्षण अधिनियम 1986, भारत सरकार को विभिन्न स्रोतों से पर्यावरण प्रदूषकों के उत्सर्जन या निस्सारण के मानक अधिकथित करने का अधिकार देता है: परन्तु ऐसे स्रोतों से पर्यावरण प्रदूषकों के उत्सर्जन या निस्सारण की गुणवत्ता या सम्मिश्रण को ध्यान में रखते हुए, भिन्न-भिन्न स्रोतों से उत्सर्जन या निस्सारण के लिये इस खण्ड के अधीन भिन्न-भिन्न मानक अधिकथित किये जा सकने का प्रावधान करता है। अतः कथन 2 सही है।

source:

https://www.drishtiias.com/hindi/daily-updates/daily-news-analysis/increasing-trap-of-plastic-pollution

https://www.drishtiias.com/hindi/daily-updates/daily-news-analysis/coastal-regulation-zone-4


59. भारत में ठोस अपशिष्ट प्रबंधन नियम, 2016 के अनुसार निम्नलिखित में से कौन-सा एक कथन सही है?

(a) अपशिष्ट उत्पादक को पाँच कोटियों में अपशिष्ट अलग-अलग करने होंगे।

(b) ये नियम केवल अधिसूचित नगरीय स्थानीय निकायों, अधिसूचित नगरों तथा सभी औद्योगिक नगरों पर ही लागू होंगे।

(c) इन नियमों में अपशिष्ट भराव स्थलों तथा अपशिष्ट प्रसंस्करण सुविधाओं के लिये सटीक और ब्यौरेवार मानदंड उपबंधित  हैं।

(d) अपशिष्ट उत्पादक के लिये यह आज्ञापक होगा कि किसी एक ज़िले में उत्पादित अपशिष्ट, किसी अन्य ज़िले में न ले जाया जाए।

उत्तर: (c)

व्याख्या: ठोस अपशिष्ट प्रबंधन, नियम 2016 के अंतर्गत, दो वर्ष के भीतर 10 लाख या उससे अधिक आबादी वाले सभी स्थानीय निकायों द्वारा अपशिष्ट प्रसंस्करण सुविधाएँ स्थापित करनी होंगी। 1 मिलियन से कम एवं  0.5 मिलियन या अधिक आबादी वाले सभी स्थानीय निकायों को या सभी स्थानीय निकायों और जनगणना शहरों द्वारा सामान्य या क्षेत्रीय अपशिष्ट भराव स्थलों को स्थापित करने के लिये, सामान्य या एकल अपशिष्ट भराव स्थलों की स्थापना का कार्य तीन वर्ष की समय सीमा में पूरा करना होगा। इसलिये अपशिष्ट भराव स्थलों और अपशिष्ट प्रसंस्करण सुविधाओं के लिये नियम  एवं विस्तृत मानदंड प्रदान करते हैं। अतः विकल्प C सही है।

प्रदूषणकर्त्ता के कर्त्तव्यों का निर्धारण करते हुए सर्वप्रथम यह कहा गया है कि प्रदूषणकर्त्ता सम्पूर्ण अपशिष्ट को तीन प्रकारों यथा जैव निम्नीकरणीय, गैर-जैव निम्नीकरणीय एवं घरेलू खतरनाक अपशिष्टों के रूप में वर्गीकृत करके इन्हें अलग-अलग डब्बों में रखकर स्थानीय निकाय द्वारा निर्धारित अपशिष्ट संग्रहकर्त्ता को ही देंगे । अतः विकल्प A सही नही है।

source:

https://www.drishtiias.com/hindi/daily-updates/daily-news-editorials/waste-management-and-new-legal-provisions


60. निम्नलिखित कथनों पर विचार कीजिये:

औद्योगिक रोज़गार (स्थायी आदेश) केंद्रीय (संशोधन) नियम, 2018 के अनुसार

1. यदि नियत अवधि रोज़गार के लिये नियमों के कार्यान्वित किया जाता है, तो फर्म/कंपनियों के लिये कामगारों की छँटनी करना अपेक्षाकृत आसान हो जाता है
2. अस्थायी कामगारों के मामलों में रोज़गार समाप्त करने के लिये कोई नोटिस देना आवश्यक नहीं होगा

उपर्युक्त में से कौन-सा/से कथन सही है/हैं?

(a) केवल 1

(b) केवल 2

(c) 1 और 2 दोनों

(d) न तो 1, न ही 2

उत्तर: (c)

व्याख्या: सरकार ने औद्योगिक रोज़गार (स्थायी आदेश) अधिनियम, 1946 में संशोधन के माध्यम से सभी क्षेत्रों के लिये रोजगार की निश्चित अवधि को अधिसूचित किया है। इसने बिचौलियों की भूमिका को कम करने के साथ-साथ सभी क्षेत्रों के लिये निश्चित अवधि के रोज़गार से कंपनियों के लिये कर्मचारियों नियुक्ति और बर्खास्तगी आसान हो जाएगी। अतः कथन 1 सही है। अस्थाई कामगार चाहे वह मासिक पारिश्रमिक पर हो, साप्ताहिक पारिश्रमिक पर हो या कार्य आधारित पारिश्रमिक पर हो या फिर परिवीक्षाधीन या बदली कामगार हो, इन सभी मामलों में रोज़गार समाप्ति की कोई सूचना देना आवश्यक नहीं होगी। अतः कथन 2 सही है।


61. सेवा क्षेत्र उपागम किससे कार्यक्षेत्र के अधीन कार्यान्वित किया गया था?

(a) एकीकृत ग्रामीण विकास कार्यक्रम

(b) अग्रणी बैंक योजना (लीड बैंक स्कीम)

(c) महात्मा गांधी राष्ट्रीय ग्रामीण रोज़गार गारंटी योजना

(d) राष्ट्रीय कौशल विकास मिशन

उत्तर: (b)

व्याख्या: अग्रणी बैंक योजना के तहत, ग्रामीण और अर्ध-शहरी क्षेत्रों के नियोजित और व्यवस्थित विकास के लिये सेवा क्षेत्र दृष्टिकोण 1989 में पेश किया गया था। इसके अंतर्गत ग्रामीण और अर्ध-शहरी क्षेत्र में प्रत्येक बैंक शाखा को 15 से 25 ग्रामों की सेवा के लिये नामित किया गया था निर्धारित बैंक शाखाएँ अपने सेवा क्षेत्र में बैंक ऋण की ज़रूरतों को पूरा करने के लिये ज़िम्मेदार थी।

Source: 

Covered in Drishti: DLP Book for UPPCS Mains/Indian Economy-I/Chapter: Bankng & Monetary Policy


62. भारत में गौण खनिज के प्रबंधन के संदर्भ में, निम्नलिखित कथनों पर विचार कीजिये:

1. इस देश में विद्यमान विधि के अनुसार रेत एक ‘गौण खनिज’ है।
2. गौण खनिजों के खनन पट्टे प्रदान करने की शक्ति राज्य सरकारों के पास है, किंतु गौण खनिजों को प्रदान करने से संबंधित नियमों को बनाने के बारे में शक्तियाँ केंद्र सरकार के पास हैं।
3. गौण खनिजों के अवैध खनन को रोकने के लिये नियम बनाने की शक्ति राज्य सरकारों के पास है।

उपर्युक्त में से कौन-सा/से कथन सही है/हैं?

(a) केवल 1 और 3

(b) केवल 2 और 3

(c) केवल 3         

(d) 1, 2 और 3

उत्तर: (a)

व्याख्या: रेत, खान और खनिज (विकास और विनियमन) अधिनियम, 1957 (MMDR अधिनियम) की धारा 3(E) के तहत परिभाषित एक गौण खनिज है। अतः कथन 1 सही है। MMDR अधिनियम की धारा 15 राज्य सरकारों को गौण खनिजों के संबंध में और उनसे जुड़े उद्देश्यों के लिये खनिज रियायतों के अनुदान को विनियमित करने हेतु नियम बनाने का अधिकार देती है। अतः गौण खनिजों के लिये खनिज रियायतें प्रदान करने का विनियमन राज्य सरकारों के विधायी और प्रशासनिक प्राधिकार क्षेत्र के अंतर्गत आता है। MMDR अधिनियम की धारा 15 द्वारा प्राप्त शक्तियों के तहत राज्य सरकारों ने अपने गौण खनिज रियायत नियम बनाएँ हैं। अतः कथन 2 गलत है। MMDR अधिनियम, 1957 की धारा 23(C) राज्य सरकारों को खनिजों के अवैध खनन, परिवहन और भंडारण पर अंकुश लगाने तथा उससे जुड़े उद्देश्यों के लिये नियम बनाने का अधिकार देती है। अतः अवैध खनन का नियंत्रण राज्य सरकारों के विधायी और प्रशासनिक अधिकार क्षेत्र में है। अतः कथन 3 सही है।


63. निम्नलिखित कथनों पर विचार कीजिये:

1. भारत का अधिकांश विदेशी ऋण सरकारी सत्वों के ऋणी होने के द्वारा है।
2. भारत का सारा विदेशी ऋण US डॉलर के मूल्यवर्ग में है।

उपर्युक्त में से कौन-सा/से कथन सही है/हैं?

(a) केवल 1

(b) केवल 2

(c) 1 और 2 दोनों

(d) न तो 1, न ही 2

उत्तर: (d)

व्याख्या: अमेरिकी डॉलर मूल्यवर्गित ऋण भारत के बाह्य ऋण का सबसे बड़ा घटक है, इसके बाद भारतीय रुपया, SDR, येन और यूरो आते हैं। अतः कथन 2 सही नहीं है। वाणिज्यिक उधार भारत के बाह्य ऋण का सबसे बड़ा घटक बना हुआ है जिसके बाद NRI जमा और अल्पावधि ऋण आते हैं। अतः कथन 1 सही नहीं है।


64. भारत में किसी वाणिज्यिक बैंक की परिसंपत्ति में निम्नलिखित में से क्या शामिल नहीं है?

(a) अग्रिम

(b) जमा

(c) निवेश

(d) मांग तथा अल्प सूचना मुद्रा (मनी ऐट कॉल एंड शॉर्ट नोटिस)

उत्तर: (b)

व्याख्या: जमा अपने आप में जमाकर्त्ता के लिये बैंक द्वारा देय देनदारी है। बैंक जमा इस देनदारी को संदर्भित करता है न कि वास्तविक कोष की परिसंपत्तियों को। अतः विकल्प B सही उत्तर है।


65. भारत के संदर्भ में, मुद्रा संकट के जोखिम को कम करने में निम्नलिखित में से किस/किन कारक/कारकों का योगदान है?

1. भारत के IT सेक्टर के विदेशी मुद्रा अर्जन का
2. सरकारी व्यय के बढ़ने का
3. विदेशस्थ भारतीयों द्वारा भेजे गए धन का

नीचे दिये गए कूट का प्रयोग कर सही उत्तर चुनिये:

(a) केवल 1

(b) केवल 1 और 3

(c) केवल 2

(d) 1, 2 और 3

उत्तर: (b)

व्याख्या: किसी देश की मुद्रा के मूल्य में गिरावट से मुद्रा संकट उत्पन्न होता है। मूल्य में यह गिरावट विनिमय दरों में अस्थिरता पैदा करके एक अर्थव्यवस्था को नकारात्मक रूप से प्रभावित करती है, जिसका अर्थ है कि एक निश्चित मुद्रा की एक इकाई अब दूसरी मुद्रा में जितना उपयोग करती है उतना क्रय नहीं करती।

विदेशी मुद्रा अर्जन और विप्रेषण रुपए की मज़बूती में योगदान करते हैं। अतः कथन 1 और 3 सही हैं।

सरकारी व्यय में वृद्धि से मुद्रा के मूल्य पर कोई प्रभाव नहीं पड़ेगा। अतः कथन 2 सही नहीं है।


66. निम्नलिखित में से किस एक का यह सुझाव था कि राज्यपाल को उस राज्य के बाहर का एक प्रतिष्ठित व्यक्ति होना चाहिये और उसे एक ऐसा तटस्थ व्यक्ति होना चाहिये जिसके गहन राजनीतिक जुड़ाव न हों या उसने हाल के पिछले वर्षों में राजनीति में भाग नहीं लिया हो?

(a) पहला प्रशासनिक सुधार आयोग (1966)

(b) राजमन्नार समिति (1969)

(c) सरकारिया आयोग (1983)

(d) संविधान के कार्यचालन की समीक्षा हेतु राष्ट्रीय आयोग (2000)

उत्तर: (c)

व्याख्या: राज्यपालों के चयन के संबंध में यह सुझाव सरकारिया आयोग ने दिया था।

source:

https://www.drishtiias.com/hindi/loksabha-rajyasabha-discussions/karnataka-election


67. पंजीकृत विदेशी पोर्टफोलियो निवेशकों द्वारा उन विदेशी निवेशकों को, जो स्वयं को सीधे पंजीकृत कराए बिना भारतीय स्टॉक बाज़ार का हिस्सा चाहते हैं, निम्नलिखित में से क्या जारी किया जाता है?

(a) जमा प्रमाण-पत्र

(b) वाणिज्यिक पत्र

(c) वचन-पत्र (प्रॉमिसरी नोट)

(d) सहभागिता पत्र (पार्टिसिपेटरी नोट)

उत्तर: (d)

व्याख्या: सहभागिता पत्र [पार्टिसिपेटरी नोट्स (पी-नोट्स)] एक प्रकार के वित्तीय साधन हैं जो कि पंजीकृत संस्थागत निवेशक द्वारा विदेशी निवेशक को जारी किये जाते हैं एवं भारत में वित्तीय नियामक, यानी भारतीय सुरक्षा और विनिमय बोर्ड (SEBI) के साथ खुद को पंजीकृत किये बिना भारतीय शेयर बाज़ार में निवेश करने का इरादा रखता है।

source:

अंक - जून 2019 पृष्ठ संख्या- 45


68. निम्नलिखित कथनों पर विचार कीजिये:

1. विधि के अनुसार, प्रतिपूरक वनीकरण कोष प्रबंधन एवं योजना प्राधिकरण, राष्ट्रीय तथा राज्य, दोनों स्तरों पर होते हैं।
2. प्रतिपूरक वनीकरण निधि अधिनियम, 2016 के अधीन चलाए गए प्रतिपूरक वनीकरण कार्यक्रमों में लोगों की सहभागिता अनिवार्य (मैंडेटरि) है।

उपर्युक्त में से कौन-सा/से कथन सही है/हैं?

(a) केवल 1

(b) केवल 2

(c) 1 और 2 दोनों

(d) न तो 1, न ही 2

उत्तर: (a)

व्याख्या: प्रतिपूरक वनीकरण कोष अधिनियम 2016, भारत के सार्वजनिक खाते के तहत राष्ट्रीय प्रतिपूरक वनीकरण कोष और प्रत्येक राज्य के सार्वजनिक खाते के तहत एक राज्य प्रतिपूरक वनीकरण कोष की स्थापना करता है। इन निधियों का प्रबंधन राष्ट्रीय और राज्य प्रत्येक स्तर पर प्रतिपूरक वनीकरण कोष प्रबंधन एवं योजना प्राधिकरणों द्वारा किया जाता है जिन्हें अधिनियम के अंतर्गत स्थापित किया गया है। हालाँकि, अधिनियम के अंतर्गत कार्यक्रमों के संचालन में लोगों की अनिवार्य सहभागिता का अधिनियम में कोई उल्लेख नहीं है। अत: विकल्प a सही है।

source:

https://www.drishtiias.com/hindi/mains-practice-question/question-858


69. भारत में दूरसंचार, बीमा, विद्युत् आदि जैसे क्षेत्रकों में स्वतंत्र नियामकों का पुनरीक्षण निम्नलिखित में से कौन करते/करती हैं?

1. संसद द्वारा गठित तदर्थ समितियाँ
2. संसदीय विभाग संबंधी स्थायी समितियाँ
3. वित्त आयोग
4. वित्तीय क्षेत्र विधायी सुधार आयोग
5. नीति (NITI) आयोग

नीचे दिये गए कूट का प्रयोग कर सही उत्तर चुनिये:

(a) 1 और 2   

(b) 1, 3 और 4

(c) 3, 4 और 5

(d) 2 और 5

उत्तर: (a)

व्याख्या: भारत में 24 संसदीय विभाग संबंधी स्थायी समितियाँ हैं जिनमें संसद के दोनों सदनों से निर्धारित सदस्यों को शामिल किया जाता है। ये समितियाँ मंत्रालय विशिष्ट होती हैं और अपने संबंधित विभागों के भीतर नियामकों के कार्यों का पुनरीक्षण कर सकती हैं। उदाहरण के लिये, अगस्त 2012 में, ऊर्जा संबंधी स्थायी समिति ने केंद्रीय विद्युत नियामक आयोग के कार्यों पर अपनी रिपोर्ट पेश की थी। वर्ष  2011 में, सूचना प्रौद्योगिकी की स्थायी समिति ने ट्राई (TRAI) के कार्यों का पुनरीक्षण किया था। संसद तदर्थ समितियों की स्थापना भी कर सकती है जो नियामकों के कार्यों का पुनरीक्षण कर सकती हैं। उदाहरण के लिये, 2जी स्पेक्ट्रम के आवंटन पर संयुक्त संसदीय समिति (JPC) के संदर्भ की शर्तों में स्पेक्ट्रम मूल्य निर्धारण और दूरसंचार लाइसेंस देने की नीति का पुनरीक्षण आदि। तदर्थ समितियों के माध्यम से संसदीय निरीक्षण का एक और उदाहरण स्टॉक मार्केट भ्रष्टाचार पर संयुक्त संसदीय समिति द्वारा SEBI और RBI के कार्यों की समीक्षा के रूप में देखने को मिलता है। अत: विकल्प A सही है।

Source: https://www.prsindia.org/sites/default/files/parliament_or_policy_pdfs/1370586800_Parliamentary%20Oversight%20of%20Regulators.pdf


70. भारत की पंचवर्षीय योजनाओं के संदर्भ में, निम्नलिखित में से कौन-सा/से कथन सही है/हैं?

1. दूसरी पंचवर्षीय योजना से बुनियादी तथा पूंजीगत वस्तु उद्योगों के प्रतिस्थापन की दिशा में निश्चयात्मक ज़ोर दिया गया।
2. चौथी पंचवर्षीय योजना में संपत्ति तथा आर्थिक शक्ति के बढ़ते संकेंद्रण की पूर्व प्रवृत्ति के सुधार का उद्देश्य अपनाया गया।
3. पाँचवीं पंचवर्षीय योजना में, पहली बार, वित्तीय क्षेत्रक को योजना के अभिन्न अंग के रूप में शामिल किया गया।

नीचे दिये गये कूट का प्रयोग सही उत्तर चुनिये।

(a) केवल 1 और 2

(b) केवल 2

(c) केवल 3         

(d) 1, 2 और 3

उत्तर: (a)

व्याख्या: दूसरी पंचवर्षीय योजना का घोषित उद्देश्य तीव्र औद्योगीकरण था जिसमें बुनियादी और पूंजीगत वस्तु उद्योगों के प्रतिस्थापन की दिशा में निश्चयात्मक ज़ोर दिया गया था - जिसका उद्देश्य समाज के एक समाजवादी प्रतिरूप की स्थापना करना था।

चौथी पंचवर्षीय योजना में सामाजिक समानता और न्याय प्राप्त करने के लिये आय, संपत्ति और आर्थिक शक्ति के संकेंद्रण की पूर्व प्रवृत्ति के सुधार पर बल दिया गया।

पाँचवीं पंचवर्षीय योजना का लक्ष्य गरीबी को दूर करना और आत्मनिर्भरता हासिल करना था।जबकि आठवीं पंचवर्षीय योजना ने वित्तीय क्षेत्र के सुधारों पर ध्यान केंद्रित किया।

Source: http://planningcommission.nic.in/plans/planrel/fiveyr/4th/4ppre.htm


71. एशियाई आधारिक-संरचना निवेश बैंक ख्एशियन इंप्रास्ट्रक्चर इंवेस्टमेंट बैंक (AIIB)] के संदर्भ में, निम्नलिखित कथनों पर विचार कीजिये:

1. AIIB के 80 से अधिक सदस्य राष्ट्र हैं।
2. AIIB में भारत सबसे बड़ा शेयरधारक है।
3. AIIB में एशिया से बाहर का कोई सदस्य नहीं है।

उपर्युत्त कथनों में से कौन-सा/से सही है/हैं?

(a) केवल 1         

(b) केवल 2 और 3

(c) केवल 1 और 3

(d) 1, 2 और 3

उत्तर: (a)

व्याख्या: AIIB एक बहुपक्षीय विकास बैंक है जिसके वर्तमान में 90 से अधिक अनुमोदित सदस्य हैं। ऑस्ट्रेलिया, न्यूज़ीलैंड, कतर, कनाडा, साइप्रस आदि इसके गैर क्षेत्रीय सदस्य हैं। अतः कथन 3 सही नहीं है। भारत AIIB का दूसरा सबसे बड़ा शेयरधारक है। (व्यापार मानक) और भारत AIIB का सबसे बड़ा ऋणकर्त्ता भी है।

source:

https://www.drishtiias.com/hindi/daily-updates/daily-news-analysis/aiib-to-invest-200-mn-in-infra-fund

https://www.drishtiias.com/hindi/mains-practice-question/question-1813


72. हाल ही में, भारतीय बैंकों और वित्तीय संस्थाओं द्वारा अंतर-ऋणदाता करार (इंटर-क्रेडिटर एंग्रीमेंट) पर हस्ताक्षर करने का क्या उद्देश्य था?

(a) भारत सरकार के राजकोषीय घाटे और चालू लेखा घाटे के वर्षानुवर्ष पड़ने वाले भार को कम करना

(b) केंद्रीय और राज्य सरकारों की आधारिक-संरचना परियोजनाओं को संबल प्रदान करना

(c) `50 करोड़ या अधिक के ऋणों के आवेदनों के मामले में स्वतंत्र नियामक के रूप में कार्य करना

(d) `50 करोड़ या अधिक की दबावयुक्त परिसंपत्तियों (स्ट्रेस्ड ऐसेट्स) का, जो सह-संघ उधारी (कॉन्सॉर्टियम लेंडिंग) के अंतर्गत हैं, अधिक तेज़ी से समाधान करने का लक्ष्य रखना

उत्तर: (d)

व्याख्या: अंतर-ऋणदाता करार (इंटर-क्रेडिटर एंग्रीमेंट) पर हस्ताक्षर करने का उद्देश्य ₹50 करोड़ या उससे अधिक के ऋण खातों का समाधान करना है जो ऋणदाताओं के एक समूह के नियंत्रण में हैं। यह अशोध्य ऋणों के समाधान में आ रही बाधाओं को दूर करने के लिये सरकार द्वारा अनुमोदित "सशक्त" योजना का हिस्सा है। अंतर-ऋणदाता करार, सुनील मेहता समिति की एक सिफारिश है जिसका उद्देश्य अशोध्य ऋणों से निपटने के प्रयास के रूप में दबावयुक्त परिसंपत्तियों के समाधान को गति देना है।

source:

https://www.drishtiias.com/hindi/daily-updates/daily-news-analysis/banks-agree-to-resolve-stressed-assets-quickly


73. सार्वजिनक क्षेत्रक बैंकों के अध्यक्षों का चयन कौन करता है?

(a) बैंक बोर्ड ब्यूरो

(b) भारतीय रिज़र्व बैंक

(c) केंद्रीय वित्त मंत्रालय

(d) संबंधित बैंक का प्रबंधन

उत्तर: (a)

व्याख्या: बैंक बोर्ड ब्यूरो सार्वजनिक क्षेत्रक बैंकों और वित्तीय संस्थानों में निदेशक मंडल के चयन और नियुक्ति के लिये ज़िम्मेदार है।

source:

https://www.drishtiias.com/hindi/daily-updates/daily-news-analysis/questions-over-capacity-of-banks-board-bureau


74. निम्नलिखित कथनों पर विचार कीजिये:

1. पेट्रोलियम और प्राकृतिक गैस नियामक बोर्ड ख्पेट्रोलियम एंड नेचुरल गैस रेग्युलेटरि बोर्ड (PNGRB)] भारत सरकार द्वारा स्थापित प्रथम नियामक निकाय है।
2. PNGRB का एक कार्य गैस के लिये प्रतियोगी बाज़ारों को सुनिश्चित करना है।
3. PNGRB के निर्णयों के विरुद्ध अपील, विद्युत् अपील अधिकरण के समक्ष की जाती है।

उपर्युक्त में से कौन-से कथन सही हैं?

(a) केवल 1 और 2

(b) केवल 2 और 3

(c) केवल 1 और 3

(d) 1, 2 और 3

उत्तर: (b)

व्याख्या: पेट्रोलियम और प्राकृतिक गैस नियामक बोर्ड (PNGRB) का गठन पेट्रोलियम और प्राकृतिक गैस नियामक बोर्ड अधिनियम, 2006 के अंतर्गत किया गया था। ट्राई भारत की पहली स्वतंत्र नियामक संस्था है।

Source: http://www.pngrb.gov.in/about-us.html


75. संचार प्रौद्योगिकी के संदर्भ में, LTE (लॉन्ग-टर्म इवॉल्यूशन) और VoLTE (वॉइस ओवर लॉन्ग-टर्म इवॉल्यूशन) के बीच क्या अंतर है/हैं?

1. LTE को साधारणत: 3G के रूप में विपणित किया जाता है तथा VoLTE को साधारणत: उन्नत 3G के रूप में विपणित किया जाता है।
2. LTE डेटा-ओन्लि तकनीक है और VoLTE वॉइस-ओन्लि तकनीक है।

नीचे दिये गए कूट का प्रयोग कर सही उत्तर चुनिये:

(a) केवल 1

(b) केवल 2

(c) 1 और 2 दोनों

(d) न तो 1, न ही 2

उत्तर: (d)

व्याख्या: LTE ने 3G WCDMA की रेडियो तकनीक में सुधार किया और OFDM या ऑर्थोगोनल फ्रीक्वेंसी-डिवीजन मल्टीप्लेक्सिंग की शुरुआत की। ITU (अंतर्राष्ट्रीय दूरसंचार संघ) ने 4G के संदर्भ में स्पष्टता की कमी को पहचानते हुए LTE-एडवांस्ड तथा वायरलेस MAN-एडवांस्ड या WiMax 2 को 4G के रूप में निर्धारित किया है।

VoLTE मोबाइल फोन नेटवर्क द्वारा उपयोग किये जाने वाले LTE प्रोटोकॉल के लिये एक प्रोद्योगिकी अद्यतन है। LTE के अंतर्गत, दूरसंचार कंपनियों का बुनियादी ढाँचा केवल डेटा के प्रसारण की अनुमति देता है, जबकि वॉयस कॉल उनके पुराने 2जी या 3जी नेटवर्क पर ही रूट किये जाते हैं।

मूल रूप से VoLTE सिस्टम वॉयस कॉल को डेटा स्ट्रीम में परिवर्तित देता है, जिसे उसी समय डेटा कनेक्शन का उपयोग करके प्रसारित किया जाता है।

source:

https://www.drishtiias.com/hindi/daily-updates/daily-news-analysis/volte-services

https://www.drishtiias.com/hindi/mains-practice-question/question-822


76. मातृत्व लाभ (संशोधन) अधिनियम, 2017 के संबंध में, निम्नलिखित में से कौन-सा/से कथन सही है/हैं?

1. गर्भवती महिलाएँ, प्रसव-पूर्व तीन महीने और प्रसवोत्तर तीन महीने के लिये सवेतन अवकाश की हकदार हैं।
2. शिशुगृहों वाले प्रतिष्ठानों के लिये माता को प्रतिदिन कम-से-कम छह बार शिशुगृह जाने की अनुमति देना अनिवार्य होगा
3. दो बच्चों वाली महिलाओं को न्यूनीकृत हक मिलेंगे।

नीचे दिये गए कूट का प्रयोग कर सही उत्तर चुनिये:

(a) केवल 1 और 2

(b) केवल 2

(c) केवल 3

(d) 1, 2 और 3

उत्तर: (c)

व्याख्या: यह विधेयक मातृत्त्व लाभ अधिनियम,1961 में संशोधन करता है। नए प्रावधानों के अंतर्गत, अब माताएँ दिन में चार बार शिशुगृह जा सकती हैं एवं इन्हें 26 सप्ताह का मातृत्त्व लाभ प्राप्त होगा। इन 26 सप्ताहों में से प्रसव से पूर्व भी आठ सप्ताह के लिये सवैतनिक अवकाश लिया जा सकता है। हालाँकि, एक साथ पूरे 26 सप्ताह के लिये अवकाश, प्रसव के बाद ही लिया जा सकेगा। यदि महिला के दो से अधिक जीवित बच्चे हैं, तो मातृत्त्व लाभ केवल 12 सप्ताह के लिये ही प्रदान किया जाएगा ।

source:

अंक- फरवरी, 2019 पृष्ठ संख्या- 35


77. निम्नलिखित में से कौन-सा एक विश्व बैंक के ‘कारोबार सुगमता सूचकांक (ईज़ ऑफ डूइंग बिज़नेस इंडेक्स)’ का उप-सूचकांक नहीं है?

(a) कानून और व्यवस्था बनाए रखना

(b) करों का भुगतान करना

(c) संपत्ति का पंजीकरण करना

(d) निर्माण परमिट संबंधी कार्य करना

उत्तर: (a)

व्याख्या: कारोबार सुगमता सूचकांक (ईज़ ऑफ डूइंग बिज़नेस इंडेक्स)’ के उप-सूचकांकों में व्यवसाय प्रारंभ करना, स्थान प्राप्त करना (श्रम बाज़ार विनियमन, निर्माण परमिट, विद्युत् प्राप्त करना, संपत्ति का पंजीकरण), वित्त तक पहुँच (ऋण प्राप्त करना, अल्पसंख्यक निवेशकों का संरक्षण), दैनिक परिचालन गतिविधियों ( जैसे कर का भुगतान, अनुबंध लागू करना) का समाधान शामिल है।

source:

https://www.drishtiias.com/hindi/daily-updates/daily-news-analysis/ease-of-doing-business


78. भारत में निम्नलिखित में से किसमें एक महत्त्वपूर्ण विशेषता के रूप में ‘विस्तारित उत्पादक दायित्व’ आरंभ किया गया था?

(a) जैव चिकित्सा अपशिष्ट (प्रबंधन और हस्तन) नियम, 1998

(b) पुनर्चक्रित प्लास्टिक (निर्माण और उपयोग) नियम, 1999

(c) ई-अपशिष्ट (प्रबंधन और हस्तन) नियम, 2011

(d) खाद्य सुरक्षा और मानक विनियम, 2011

उत्तर: (c)

व्याख्या: ई-अपशिष्ट (प्रबंधन और हस्तन) नियम, 2011 "विस्तारित उत्पादक दायित्व" (EPR) की अवधारणा को शामिल करके विद्युत और इलेक्ट्रॉनिक उपकरणों के उत्पादकों पर ई-अपशिष्ट प्रबंधन की मुख्य ज़िम्मेदारी डालते हैं। उपयोग किये हुए अपशिष्ट उत्पादों के उपचार या निपटान के लिये उत्पादकों को महत्त्वपूर्ण  - वित्तीय या भौतिक ज़िम्मेदारी सौंपी गई है। ई-अपशिष्ट (प्रबंधन और हस्तन) नियम, 2011 को मई 2011 में अधिसूचित किया गया है एवं यह नियम वर्ष 2012 से प्रभाव में हैं।

source:

https://www.drishtiias.com/hindi/daily-updates/daily-news-analysis/cpcb-given-extended-producer-responsibility-authorisation-in-new-e-waste-rule

https://www.drishtiias.com/hindi/daily-updates/daily-news-analysis/increasing-trap-of-plastic-pollution


79. भारतीय खाद्य निगम के लिये खाद्यान्नों की आर्थिक लागत में न्यूनतम समर्थन मूल्य और किसानों को भुगतान किये गए बोनस (यदि कुछ है) के साथ-साथ और क्या शामिल है/हैं?

(a) केवल परिवहन लागत

(b) केवल ब्याज लागत

(c) प्रापण प्रासंगिक प्रभार तथा वितरण लागत

(d) प्रापण प्रासंगिक प्रभार तथा गोदामों के प्रभार

उत्तर: (c)

व्याख्या: भारतीय खाद्य निगम के लिये खाद्यान्नों की आर्थिक लागत में न्यूनतम समर्थन मूल्य और किसानों को भुगतान किये गए बोनस (यदि कुछ है) के साथ-साथ प्रापण प्रासंगिक प्रभार तथा वितरण लागत शामिल है।


80. किसी भी देश के संदर्भ में, निम्नलिखित में से किसे उस देश की सामाजिक पूंजी (सोशल कैपिटल) के भाग के रूप में समझा जाएगा?

(a) जनसंख्या में साक्षरों का अनुपात

(b) इसके भवनों, अन्य आधारिक संरचना और मशीनों का स्टॉक

(c) कार्यशील आयु समूह में जनसंख्या का आमाप

(d) समाज में आपसी भरोसे और सामंजस्य का स्तर

उत्तर: (d)

व्याख्या: सामाजिक पूंजी व्यक्तियों के मध्य आपसी संबंधों एवं सामाजिक तानावाना तथा समाज में आपसी भरोसे और सामंजस्य के स्तरों को संदर्भित करती है।

Source: https://darpg.gov.in/sites/default/files/Social_Capital9.pdf


81. भारत के संविधान के संदर्भ में, निम्नलिखित कथनों पर विचार कीजिये:

1. किसी भी केंद्रीय विधि को सांविधानिक रूप से अवैध घोषित करने की किसी भी उच्च न्यायालय की अधिकारिता नहीं होगी।
2. भारत के संविधान के किसी भी संशोधन पर भारत के उच्चतम न्यायालय द्वारा प्रश्न नहीं उठाया जा सकता।

उपर्युक्त में से कौन-सा/से कथन सही है/हैं?

(a) केवल 1

(b) केवल 2

(c) 1 और 2 दोनों

(d) न तो 1, न ही 2

उत्तर: (d)

व्याख्या:  42वे संविधान संशोधन अधिनियम,1976 ने उच्च न्यायालय की न्यायिक समीक्षा शक्तियों में  कटौती की और किसी भी केंद्रीय कानून की संवैधानिक वैधता पर विचार करने से उच्च न्यायालयों को रोक दिया। हालाँकि, 1977 के 43वें संविधान संशोधन अधिनियम ने मूल स्थिति को पुनः स्थापित कर दिया था। उच्च न्यायालय के पास संविधान के मूलभूत सिद्धांतों के विरुद्ध जाने वाले किसी भी केंद्रीय कानून को अमान्य करने की शक्ति है। उच्चतम न्यायालय भी यह शक्तियाँ रखता है। यदि कोई संवैधानिक संशोधन संविधान की मूल संरचना के विरुद्ध है, तो उच्चतम न्यायालय के पास इसे अमान्य घोषित करने की शक्ति है। इन शक्तियों का ही प्रयोग करते हुए वर्ष 2015 में, उच्चतम न्यायालय ने 99वें संवैधानिक संशोधन, 2014 तथा राष्ट्रीय न्यायिक नियुक्ति आयोग (NJAC) अधिनियम, 2014 को असंवैधानिक और शून्य घोषित कर दिया था।


82. निम्नलिखित कथनों पर विचार कीजिये:

1. क्रय शक्ति समता ख्परचेज़़िग पावर पैरिटि (PPP)] विनिमय दरों की गणना विभिन्न देशों में एकसमान वस्तुओं और सेवाओं के मूल्यों की तुलना कर की जाती है।
2. PPP डॉलर के संदर्भ में, भारत विश्व की छठी सबसे बड़ी अर्थव्यवस्था है।

उपर्युक्त में से कौन-सा/से कथन सही है/हैं?

(a) केवल 1

(b) केवल 2

(c) 1 और 2 दोनों

(d) न तो 1, न ही 2

उत्तर: (a)

व्याख्या: क्रय शक्ति समता [परचेज़िग पावर पैरिटि (PPP)] एक आर्थिक सिद्धांत है जो किसी देश की मुद्रा की एक इकाई द्वारा दूसरे देश में खरीदी जा सकने वाली वस्तुओं और सेवाओं की कुल मात्रा को मापता है। विभिन्न देशों की मुद्रा की तुलना एकसमान वस्तुओं और सेवाओं के माध्यम से करता है। देश A और B के बीच PPP देश B में वस्तुओं और सेवाओं की एक इकाई खरीदने के लिये देश B की मुद्रा की तुलना में देश A में वस्तुओं और सेवाओं की एक इकाई खरीदने के लिये देश A की आवश्यक मुद्रा की मात्रा को मापता है।


83. भारत में पिछले पाँच वर्षों में खरीफ की फसलों की खेती के संदर्भ में, निम्नलिखित कथनों पर विचार कीजिये:

1. धान की खेती के अंतर्गत क्षेत्र अधिकतम है।
2. ज्वार की खेती के अंतर्गत क्षेत्र, तिलहनों की खेती के अंतर्गत क्षेत्र की तुलना में अधिक है।
3. कपास की खेती का क्षेत्र, गन्ने की खेती के क्षेत्र की तुलना में अधिक है।
4. गन्ने की खेती के अंतर्गत क्षेत्र निरंतर घटा है।

उपर्युक्त में से कौन-से कथन सही हैं?

(a) केवल 1 और 3

(b) केवल 2, 3 और 4

(c) केवल 2 और 4

(d) 1, 2, 3 और 4

उत्तर: (a)


84. भारत द्वारा आयातित कृषि जिंसों में, पिछले पाँच वर्षों में निम्नलिखित में से किस एक का मूल्य के आधार पर अधिकतम आयात रहा है?

(a) मसाले

(b) ताज़े फल

(c) दलहन

(d) वनस्पति तेल

उत्तर: (d)

व्याख्या: अक्तूबर 2010 में दक्षिण पूर्व एशियाई देशों के साथ हस्ताक्षर किये, मलेशिया-भारत व्यापक आर्थिक सहयोग समझौते (MICECA) के तहत शुल्क में कटौती के कारण भारत में वनस्पति तेलों का आयात पिछले पाँच वर्षों में सर्वाधिक रहा है। 

Source: https://www.business-standard.com/article/markets/edible-oil-imports-jump-11-in-dec-may-rise-further-on-duty-cut-119011500879_1.html


85. राज्य-व्यवस्था के संदर्भ में, निम्नलिखित में से किस एक को आप स्वतंत्रता की सर्वाधिक उपयुक्त उत्तर के रूप में स्वीकार करेंगे?

(a) राजनीतिक शासकों की तानाशाही के विरुद्ध संरक्षण

(b) नियंत्रण का अभाव

(c) इच्छानुसार कुछ भी करने का अवसर

(d) स्वयं को पूर्णत: विकसित करने का अवसर

उत्तर: (b/d)

व्याख्या: विकल्प d सबसे उपयुक्त उत्तर प्रतीत होता है क्योंकि भारतीय संविधान की प्रस्तावना विचार, अभिव्यक्ति, धर्म, विश्वास व उपासना की स्वतंत्रता प्रदान करती है। इसकी व्याख्या राजनीति के संदर्भ में स्वतंत्रता के रूप में की जा सकती है।

Source: Preamble of india


86. भारतीय रुपए की गिरावट रोकने के लिये निम्नलिखित में से कौन-सा एक सरकार/भारतीय रिज़र्व बैंक द्वारा किया जाने वाला सर्वाधिक संभावित उपाय नहीं है?

(a) गैर-ज़रूरी वस्तुओं के आयात पर नियंत्रण और निर्यात को प्रोत्साहन

(b) भारतीय उधारकर्त्ताओं को रुपए मूल्यवर्ग के मसाला बॉन्ड जारी करने हेतु प्रोत्साहित करना

(c) विदेशी वाणिज्यिक उधारी से संबंधित दशाओं को आसान बनाना

(d) एक प्रसरणशील मौद्रिक नीति का अनुसरण करना

उत्तर: (d)

व्याख्या: गैर-आवश्यक वस्तुओं के आयात पर नियंत्रण से डॉलर की माँग कम होगी और निर्यात को प्रोत्साहन देश में डॉलर के प्रवाह में वृद्धि करेगा एवं यह रुपए की गिरावट को नियंत्रित करने में सहायक होगा।

मसाला बाॅण्ड सीधे भारतीय मुद्रा से जुड़ा हुआ है। यदि भारतीय उधारकर्त्ता रुपए में अधिक मूल्यवर्ग के मसाला बाॅण्ड जारी करते हैं, तो इससे बाज़ार में तरलता बढ़ेगी या बाज़ार में कुछ मुद्राओं के मुकाबले रुपए के स्टॉक में वृद्धि होगी जो रुपए की स्थिति को मज़बूत करने में सहायक होगा।

विदेशी वाणिज्यिक उधारी (ECB)- यह विदेशी मुद्रा में लिया गया एक प्रकार का ऋण है, जो अनिवासी ऋणदाताओं द्वारा प्रदान किया जाता है। इस प्रकार ECB की आसान उपलब्धता विदेशी मुद्रा में अधिक ऋण प्राप्त करने में सहायक होती है इससे न केवल विदेशी मुद्रा प्रवाह में वृद्धि होगी बल्कि यह रुपए की स्थिति को मज़बूत करने में भी सहायक होगा।

विस्तारवादी मौद्रिक नीति- अर्थव्यवस्था को प्रोत्साहित करने के लिये RBI द्वारा धन की आपूर्ति में वृद्धि जैसे नीतिगत उपायों का एक समूह है। जो कि रुपए के मूल्य में परिवर्तन को प्रभावित नहीं कर सकता है।

Source: https://www.thehindu.com/business/Economy/curb-on-imports-to-bring-rupee-to-68-70-level-department-of-economic-affairs/article25019942.ece


87. निम्नलिखित कथनों पर विचार कीजिये:

‘भुगतान प्रणाली आँकड़ों के भंडारण (स्टोरेज ऑफ पेमेंट सिस्टम डेटा)’ के संबंध में भारतीय रिज़र्व बैंक के हाल का निदेश, जिसे प्रचलित रूप से डेटा डिक्टैट के रूप में जाना जाता है, भुगतान प्रणाली प्रदाताओं (पेमेंट सिस्टम प्रोवाइडर्स) को समादेशित करता है कि

1. वे यह सुनिश्चित करेंगे कि उनके द्वारा संचालित भुगतान प्रणालियों से संबंधित समग्र आँकड़े एक प्रणाली के अंतर्गत केवल भारत में भंडारित किये जाएँ
2. वे यह सुनिश्चित करेंगे कि इन प्रणालियों का स्वामित्व और संचालन सार्वजनिक क्षेत्र के उद्यम ही करें
3. वे कैलेंडर वर्ष की समाप्ति तक भारत के नियंत्रक एवं महालेखापरीक्षक को समेकित प्रणाली लेखापरीक्षा रिपोर्ट प्रस्तुत करेंगे

उपर्युक्त में से कौन-सा/से कथन सही है/हैं?

(a) केवल 1

(b) केवल 1 और 2

(c) केवल 3

(d) 1, 2 और 3

उत्तर: (a)

व्याख्या: भुगतान प्रणाली आँकड़ों के भंडारण (स्टोरेज ऑफ पेमेंट सिस्टम डेटा)’ के संबंध में भारतीय रिज़र्व बैंक के हाल का निदेश, जिसे प्रचलित रूप से डेटा डिक्टैट के रूप में जाना जाता है, भुगतान प्रणाली प्रदाताओं (पेमेंट सिस्टम प्रोवाइडर्स) को समादेशित करता है कि वे यह सुनिश्चित करेंगे कि उनके द्वारा संचालित भुगतान प्रणालियों से संबंधित समग्र आँकड़े एक प्रणाली के अंतर्गत केवल भारत में भंडारित किये जाएँ। इसके अलावा, भुगतान प्रणाली प्रदाताओं को भुगतान प्रणाली की लेखापरीक्षा रिपोर्ट (SAR) भी प्रस्तुत करनी होगी और इस संबंध में लेखापरीक्षा का कार्य CERT-IN के पैनलबद्ध लेखापरीक्षकों द्वारा किया जाएगा।

Source:

 RBI Website: https://www.rbi.org.in/scripts/NotificationUser.aspx?Id=11244


88. निम्नलिखित में से किसने अपने नागरिकों के लिये दत्त संरक्षण (डेटा प्रोटेक्शन) और प्राइवेसी के लिये ‘सामान्य दत्त संरक्षण विनियमन (जनरल डेटा प्रोटेक्शन रेगुलेशन)’ नामक एक कानून अप्रैल 2016 में अपनाया और उसका 25 मई, 2018 से कार्यान्वयन शुरू किया?

(a) ऑस्ट्रेलिया

(b) कनाडा

(c) यूरोपीय संघ (यूरोपियन यूनियन)

(d) संयुक्त राज्य अमेरिका

उत्तर: (c)

व्याख्या: सामान्य दत्त संरक्षण विनियमन (जनरल डेटा प्रोटेक्शन रेगुलेशन) सभी 28 यूरोपीय संघ के देशों में दत्त संरक्षण (डेटा प्रोटेक्शन) कानून का मानकीकरण करता है और व्यक्तिगत रूप से पहचान योग्य जानकारी (PII) को नियंत्रित करने तथा संसाधित करने पर सख्त नए नियम लागू करता है। यह यूरोपीय संघ के निवासियों को नियंत्रण पुनः उपलब्ध कराकर व्यक्तिगत डेटा की सुरक्षा और दत्त संरक्षण (डेटा प्रोटेक्शन) अधिकारों का विस्तार भी करता है। GDPR ने वर्ष 1995 के EU दत्त संरक्षण (डेटा प्रोटेक्शन) निर्देशों की जगह ली और  25 मई, 2018 से कार्यान्वयन शुरू किया।

source:

https://www.drishtiias.com/hindi/daily-updates/daily-news-analysis/eu-new-data-protection-rules-come-into-effect


89. हाल ही में भारत ने निम्नलिखित में से किस देश के साथ ‘नाभिकीय क्षेत्र में सहयोग क्षेत्राें के प्राथमिकीकरण और कार्यान्वयन हेतु कार्य योजना’ नामक सौदे पर हस्ताक्षर किये हैं?

(a) जापान

(b) रूस

(c) यूनाइटेड किंगडम

(d) संयुक्त राज्य अमेरिका

उत्तर: (b)

व्याख्या: रूसी के राष्ट्रपति व्लादिमीर पुतिन की भारत यात्रा के दौरान, भारत और रूस द्वारा संयुक्त रूप से चिह्नित नाभिकीय क्षेत्र में सहयोग क्षेत्राें के प्राथमिकीकरण और कार्यान्वयन हेतु कार्य योजना नामक सौदे पर 5 अक्तूबर, 2018 को नई दिल्ली में हस्ताक्षर किये गए थे।

source:

https://www.drishtiias.com/hindi/daily-updates/daily-news-analysis/India-Russia-Summit-2018


90. किसी अर्थव्यवस्था में मुद्रा गुणक (मनी मल्टिप्लायर) निम्नलिखित में से किस एक के साथ-साथ बढ़ता है?

(a) आरक्षित नकदी (कैश रिज़र्व) अनुपात में वृद्धि

(b) जनता की बैंकिंग आदतों में वृद्धि

(c) सांविधिक नकदी अनुपात में वृद्धि

(d) देश की जनसंख्या में वृद्धि

उत्तर: (b)

व्याख्या: मुद्रा गुणक (मनी मल्टिप्लायर) ब्रॉड मनी (M3मुद्रा) की अधिकतम राशि है जो वाणिज्यिक बैंकों द्वारा निश्चित राशि हेतु, मूल नकदी एवं आरक्षित अनुपात के लिये उत्पन्न  की जाती है। इसे सीधे तौर पर यह भी कहा जा सकता है कि रुपए के प्रत्येक भंडार से बैंकिंग प्रणाली जो अधिकतम धनराशि उत्पन्न करती है, उसे मुद्रा गुणक कहते है।

इसलिये यदि वित्तीय समावेशन या बैंकिंग आदतों में वृद्धि होगी तो इससे ऋण, परिदाय आदि के रूप में उपयोग में आने वाले धन से मुद्रा का प्रसार होगा जिसके परिणामस्वरूप आर्थिक गतिविधि में वृद्धि होगी। यह प्रत्यक्ष रूप से किसी अर्थव्यवस्था में मुद्रा गुणक (मनी मल्टिप्लायर) को प्रोत्साहित करने में सहायक होगा । नकद आरक्षित अनुपात (CRR) में कमी के कारण भी मुद्रा गुणक में वृद्धि होती है।

source:

https://www.drishtiias.com/hindi/daily-updates/daily-news-editorials/slowdown-in-indian-economy


91. मनोरंजन हेतु डिजिटल प्रौद्योगिकियों के संदर्भ में, निम्नलिखित कथनों पर विचार कीजिये:

1. संवर्द्धित वास्तविकता ख्ऑगमेंटेड रिएलिटि (AR)] में एक छँ वातावरण सृजित हो जाता है और भौतिक संसार पूरी तरह बहिष्कृत हो जाता है।
2. आभासी वास्तविकता ख्वर्चुअल रिएलिटि (VR), में कम्प्यूटर द्वारा सृजित प्रतिमाएँ वास्तविक जीवन की वस्तुओं या परिवेशों पर प्रक्षेपित हो जाती हैं।
3. AR व्यक्तियों को संसार में विद्यमान रहने देता है और स्मार्ट-फोन या PC के कैमरे का उपयोग कर अनुभव को उन्नत करता है।
4. VR संसार को पृथक् कर देता है और व्यक्ति को एक अलग धरातल पर ले जाकर उसे पूर्ण निमग्नता का अनुभव प्रदान करता है।

उपर्युक्त में से कौन-सा/से कथन सही है/

(a) केवल 1 और 2

(b) 3 और 4

(c) 1, 2 और 3

(d) केवल 4

उत्तर: (b)

व्याख्या:  आभासी वास्तविकता [वर्चुअल रिएलिटि (VR)] उपयोगकर्त्ताओं को पूरी तरह से कृत्रिम डिजिटल वातावरण में ले जाकर उसे पूर्ण निमग्नता का अनुभव प्रदान करता है।

संवर्द्धित वास्तविकता [ऑगमेंटेड रिएलिटि (AR)] वास्तविक दुनिया के वातावरण में आभासी वस्तुओं को प्रक्षेपित करती है।

मिश्रित वास्तविकता (MR) आभासी वस्तुओं न केवल प्रक्षेपित करती है बल्कि इन्हें वास्तविक दुनिया में आभासी रूप से स्थापित भी करती है।


92. शब्द ‘डेनिसोवन (Denisovan)’ कभी-कभी समाचार माध्यमों में किस संदर्भ में आता है?

(a) एक प्रकार के डायनासोर का जीवाश्म

(b) एक आदिमानव जाति (स्पीशीज़)

(c) पूर्वोत्तर भारत में प्राप्त एक गुफा तंत्र

(d) भारतीय उपमहाद्वीप के इतिहास में एक भूवैज्ञानिक कल्प

उत्तर: (b)

व्याख्या: पहली बार, वैज्ञानिकों को साइबेरिया के बाहर डेनिसोवन (Denisovan) के जीवाश्म प्राप्त हुए हैं जो कि आदिमानवों की एक विलुप्त जाति (स्पीशीज़) है।

डेनिसोवन (Denisovan) होमिनिन्स का एक विलुप्त समूह है जो निएंडरथल के करीबी वंशज थे। इनकी पहचान मुख्य रूप से साइबेरिया में डेनिसोवा गुफा से प्राप्त जीवाश्मों एवं एशियाई लोगों के DNA से प्राप्त आनुवंशिक साक्ष्यों से की जाती है।

source:

https://www.drishtiias.com/hindi/daily-updates/prelims-facts/prelims-facts-04-05-2019#1


93. विज्ञान में हुए अभिनव विकासों के संदर्भ में, निम्नलिखित में से कौन-सा एक कथन सही नहीं है?

(a) विभिन्न जातियों की कोशिकाओं से लिये गए DNA के खंडों को जोड़कर प्रकार्यात्मक गुणसूत्र रचे जा सकते हैं।

(b) प्रयोगशालाओं में कृत्रिम प्रकार्यात्मक DNA के हिस्से रचे जा सकते हैं।

(c) किसी जंतु कोशिका से निकाले गए DNA के किसी हिस्से को जीवित कोशिका से बाहर, प्रयोगशाला में, प्रतिकृत कराया जा सकता है।

(d) पादपों और जंतुओं से निकाली गई कोशिकाओं में प्रयोगशाला की पेट्री डिश में कोशिका विभाजन कराया जा सकता है।

उत्तर: (a*)

व्याख्या: कृत्रिम रूप से जीवन निर्माण की दिशा में एक प्रमुख कदम के रूप में अमेरिकी शोधकर्त्ताओं ने एक सजीव जीव को विकसित किया है जो प्राकृतिक और कृत्रिम DNA दोनों को शामिल करता है एवं पूर्णतः नवीन कृत्रिम प्रकार्यात्मक DNA के हिस्से को रचने में सक्षम है। अत: विकल्प (B) सही है। द्वि-कुंडलित DNA की एक विस्तृत विविधता को पुनः संयोजक DNA प्रतिकृति प्रणाली में बड़े पैमाने पर दोहराया जाता है जिसमें सात टी 4 बैक्टीरियोफेज़  DNA प्रतिकृति जीन (32, 41, 43, 44, 62, 45 और 61) द्वारा निर्दिष्ट शुद्ध प्रोटीन होते हैं। अतः विकल्प (c) सही है। सूक्ष्मप्रवर्धन के माध्यम से पौधे को प्रयोगशाला में विकसित किया जा सकता है और क्लैमाइडोमनास कोशिकाओं को प्रकाश विविधिकरण के माध्यम से जीवाणुओं के संवर्धन में दोहराया जा सकता है। अत: विकल्प (D) सही है।


94. निम्नलिखित कथनों पर विचार कीजिये:

डिजिटल हस्ताक्षर

1. एक ऐसा इलेक्ट्रॉनिक अभिलेख है, जो इसे जारी करने वाले प्रमाणन प्राधिकारी की पहचान करता है।
2. इंटरनेट पर सूचना या सर्वर तक पहुँच के लिये किसी व्यक्ति की पहचान के प्रमाण के रूप में प्रयुक्त होता है।
3. इलेक्ट्रॉनिक दस्तावेज़ पर हस्ताक्षर करने की एक इलेक्ट्रॉनिक पद्धति है और सुनिश्चित करता है कि मूल अंश अपरिवर्तित है।

उपर्युक्त में से कौन-सा/से कथन सही है/हैं?

(a) केवल 1

(b) केवल 2 और 3

(c) केवल 3

(d) 1, 2 और 3

उत्तर: (c/d)

व्याख्या: यदि हम डिजिटल हस्ताक्षर को डिजिटल हस्ताक्षर प्रमाणपत्र के समान मानते हैं तो विकल्प d सही उत्तर होना चाहिये। प्रमाण पत्र में उपयोकर्त्ता की पहचान (नाम, पिन कोड, देश, ईमेल पता, प्रमाण पत्र जारी करने की तिथि और प्रमाणन प्राधिकारी का नाम) के बारे में सूचना होती है। हालाँकि, अगर उन्हें पृथक माना जाता है, तो उत्तर होना C होना चाहिये।


95. परिधेय प्रौद्योगिकी (विअरेबल टेक्नोलॉजी) के संदर्भ में, परिधेय उपकरणों द्वारा निम्नलिखित में से कौन-सा/से कार्य निष्पन्न किया जा सकता है/ किये जा सकते हैं?

1. किसी व्यक्ति का अवस्थान (लोकेशन) निर्धारण
2. किसी व्यक्ति का निद्रा मॉनीटरन
3. श्रवण दोषयुक्त व्यक्ति की सहायता

नीचे दिये गए कूट का प्रयोग कर सही उत्तर चुनिये।

(a) केवल 1

(b) केवल 2 और 3

(c) केवल 3

(d) 1, 2 और 3

उत्तर: (d)

व्याख्या: हाल के दिनों में, कानों में पहनने योग्य वाद्य उपकरणों से संबंधित परिधेय प्रौद्योगिकी  (विअरेबल टेक्नोलॉजी) में एक नया उछाल देखने को मिला है। परिधेय प्रौद्योगिकी इलेक्ट्रॉनिक परिधेय उपकरणों का एक समूह है जिसे सहायक उपकरण के रूप में पहना जा सकता है इन्हें विविध माध्यमों जैसे; वस्त्रों में प्रयुक्त कर, उपयोगकर्त्ता के शरीर में प्रत्यारोपित कर, या त्वचा पर टैटू आदि, के द्वारा उपयोग में लाया जा सकता है।

source:

https://www.drishtiias.com/hindi/daily-updates/daily-news-analysis/prelims-fact-20-12-2017


96. ‘RNA अंतर्क्षेप [RNA इंटरपेरेंस (RNAi)]’ प्रौद्योगिकी ने पिछले कुछ वर्षों में लोकप्रियता हासिल कर ली है। क्यों?

1. यह जीन अनभिव्यक्तिकरण (जीन साइलेंसिंग) रोगोपचारों के विकास में प्रयुक्त होता है।
2. इसे कैंसर की चिकित्सा में रोगोपचार विकसित करने हेतु प्रयुक्त किया जा सकता है।
3. इसे हॉर्मोन प्रतिस्थापन रोगोपचार विकसित करने हेतु प्रयुक्त किया जा सकता है।
4. इसे ऐसी फसल पादपों को उगाने के लिये प्रयुक्त किया जा सकता है, जो विषाणु रोगजनकों के लिये प्रतिरोधी हो।

नीचे दिये गए कूट का प्रयोग कर सही उत्तर चुनिये:

(a) 1, 2 और 4 (b) 2 और 3

(c) 1 और 3     (d) केवल 1 और 4

उत्तर: (a)

व्याख्या: RNA अंतर्क्षेप [RNA इंटरपेरेंस (RNAi)] प्रौद्योगिकी, यूकेरियोटिक कोशिकाओं (स्पष्ट रूप से परिभाषित नाभिक वाली कोशिकाओं) के भीतर होने वाली नियामक प्रणाली है जो जीन की गतिविधि को नियंत्रित करती है। इसका प्रयोग जीन अनभिव्यक्तिकरण (जीन साइलेंसिंग), रोगोपचारों के विकास में किया जाता है। हेपेटाइटिस, एड्स, हंटिंगटन रोग और कैंसर सहित विभिन्न प्रकार के रोगोपचारों के विकास में RNA अंतर्क्षेप के संबंध में आज भी अनुसंधान जारी है। RNAi का उपयोग जंपिंग जीन और विषाणु आनुवंशिक तत्वों जैसे आणविक परजीवियों के विरुद्ध एक प्राकृतिक प्रतिरक्षा तंत्र के रूप में किया जाता है।

source:

https://www.drishtiias.com/hindi/daily-updates/daily-news-editorials/rna-technology-and-their-usefulness-in-context-of-india


97. हाल ही में वैज्ञानिकों ने पृथ्वी से अरबों प्रकाश-वर्ष दूर विशालकाय ‘ब्लैकहोलों’ के विलय का प्रेक्षण किया। इस प्रेक्षण का क्या महत्त्व है?

(a) ‘हिग्स बोसॉन कणों’ का अभिज्ञान हुआ।

(b) ‘गुरुत्वीय तरंगों’ का अभिज्ञान हुआ।

(c) ‘वॉर्महोल’ से होते हुए अंतरा-मंदाकिनीय अंतरिक्ष यात्रा की संभावना की पुष्टि हुई।

(d) इसने वैज्ञानिकों को ‘विलक्षणता (सिंगुलैरिटि)’ को समझना सुकर बनाया।

उत्तर: (b)

व्याख्या: जब इन प्रणालियों में दो विशालकाय ब्लैक होल अंततः लाखों वर्षों में एक साथ आते हैं, तो उनके टकराने से तीव्र गुरुत्त्वाकर्षण तरंगें उत्पन्न होंगी। लेजर इंटरफेरोमीटर ग्रेविटेशनल-वेव ऑब्जर्वेटरी (LIGO) द्वारा दो तारकीय-द्रव्यमान वाले ब्लैक होल की टक्कर से उत्पन्न गुरुत्वाकर्षण तरंगों का पहले ही पता लगाया जा चुका है। नियोजित NASA/ESA अंतरिक्ष-आधारित लेज़र इंटरफेरोमीटर स्पेस एंटीना (LISA) जैसी वेधशालाएँ विशालकाय ब्लैक-होल विलय से कम-आवृत्ति वाली गुरुत्त्वाकर्षण तरंगों के अभिज्ञान में सक्षम होंगी, जो LIGO द्वारा खोजी गई गुरुत्त्वाकर्षण तरंगों की तुलना में एक लाख गुना अधिक हैं।

Source: 

https://www.drishtiias.com/infographics/info-paper3/gravitational-waves


98. निम्नलिखित में से कौन-से, भारत में सूक्ष्मजैविक रोगजनकों में बहु-औषध प्रतिरोध के होने के कारण हैं?

1. कुछ व्यक्तियों में आनुवंशिक पूर्ववृत्ति (जेनेटिक प्रीडिस्पोजीशन) का होना
2. रोगों के उपचार के लिये प्रतिजैविकों (एंटिबॉयोटिक्स) की गलत खुराकें लेना
3. पशुधन फार्मिंग में प्रतिजैविकों का इस्तेमाल करना
4. कुछ व्यक्तियों में चिरकालिक रोगों की बहुलता होना

नीचे दिये गए कूट का प्रयोग कर सही उत्तर चुनिये:

(a) 1 और 2   

(b) केवल 2 और 3

(c) 1, 3 और 4

(d) 2, 3 और 4

उत्तर: (b)

व्याख्या: रोगाणुरोधी प्रतिरोध आमतौर पर आनुवंशिक परिवर्तनों के माध्यम से स्वाभाविक रूप से समय के साथ विकसित होता है। हालाँकि, रोगाणुरोधी का दुरुपयोग और अति प्रयोग ने इस प्रक्रिया को तेज़ करने कार्य किया है। कई स्थानों पर मानव एवं अन्य जीवों में प्रतिजैविक दवाओं का अत्यधिक उपयोग अथवा दुरुपयोग ने, जो की अक्सर पेशेवर निरीक्षण की अनुपस्थिति में होता है, इस समस्या को और भी गंभीर बना दिया है। दुरुपयोग के उदाहरणों में सर्दी और फ्लू जैसे वायरल संक्रमण में एवं पशुओं में विकास प्रमोटर के रूप में या स्वस्थ पशुओं में रोगों को रोकने के लिये इनका अत्यधिक उपयोग शामिल है।

source:

https://www.drishtiias.com/hindi/daily-updates/daily-news-analysis/amr-could-cause-10-million-deaths-un


99. प्राय: समाचारों में आने वाला Cas9 प्रोटीन क्या है?

(a) लक्ष्य-साधित जीन संपादन (टारगेटेड जीन एडिटिंग) में प्रयुक्त आण्विक कैंची

(b) रोगियों में रोगजनकों की ठीक-ठीक पहचान के लिये प्रयुक्त जैव संवेदक

(c) एक जीन जो पादपों को पीड़क-प्रतिरोधी बनाता है

(d) आनुवंशिकत: रूपांतरित फसलों में संश्लेषित होने वाला एक शाकनाशी पदार्थ

उत्तर: (a)

व्याख्या: CRISPR Cas9 "क्लस्टर्ड रेगुलरली इंटरस्पेस्ड शॉर्ट पालिंड्रोमिक रिपीट" के लिये एक संक्षिप्त रूप है। CRISPR जीनोम अभियांत्रिकी तकनीक, वैज्ञानिकों को किसी भी जीनोम के DNA को आसानी एवं सटीक रूप से संपादित करने में सक्षम बनाती है।

source:

अंक- जुलाई विज्ञान एवं प्रौद्योगिकी खंड (CRISPR)

https://www.drishtiias.com/hindi/daily-updates/daily-news-analysis/scientists-perform-first-in-body-gene-editing

https://www.drishtiias.com/hindi/daily-updates/daily-news-editorials/genetic-modification-goes-beyond-ethics


100. निम्नलिखित में से कौन-सा एक कथन सही नहीं है?

(a) यकृतशोध B विषाणु काफी कुछ HIV की तरह ही संचरित होता है।

(b) यकृतशोध C का टीका होता है, जबकि यकृतशोध B का कोई टीका नहीं होता।

(c) सार्वभौम रूप से यकृतशोध B और C विषाणुओं से संक्रमित व्यक्तियों की संख्या HIV से संक्रमित लोगों की संख्या से कई गुना अधिक है।

(d) यकृतशोध B और C विषाणुओं से संक्रमित कुछ व्यक्तियों में अनेक वर्षों तक इसके लक्षण दिखाई नहीं देते।

उत्तर: (b)

व्याख्या: यकृतशोध -B का टीका एक सुरक्षित और प्रभावी टीका है जो जन्म के समय सभी शिशुओं और 18 वर्ष तक के बच्चों के लिये अनुशंसित है। यकृतशोध -B के टीके को पूर्व में  "कैंसर-रोधी" टीके के रूप में भी जाना जाता था क्योंकि यह विश्व भर में यकृत कैंसर के प्रमुख कारण  यकृतशोध B को रोकता है।

source:

https://www.drishtiias.com/hindi/daily-updates/daily-news-analysis/health-ministry-to-immunize-healthcare-workers-with-hepatitis-b-vaccine

close
एसएमएस अलर्ट
Share Page
images-2
images-2
× Snow